TOEIC Advanced AnswerKey
TOEIC Advanced AnswerKey
ANSWER KEY
LISTENING COMPREHENSION
PART 1: PHOTOS
Exercise 1: Analyzing Photos (page 4)
Photo 1 (page 4)
Language Strategies
Possible answers:
NOUNS
passengers luggage vest airport board
travelers suitcases shorts window sign
man baggage pockets floor plant
woman tote bag suit information
people skirt glasses schedule
bags jacket hands list
VERBS
stand pull stare think
wait drag grasp worry
walk wear want
hold read arrive
carry look depart
Copyright © 2012 by Pearson Education, Inc. Permission granted to reproduce for classroom use.
LOCATIONS
The travelers are standing in front of the sign.
The clock is on the sign.
The plant is by the window.
The man’s hands are behind his back.
The man’s hands are in his pockets.
The windows are behind the sign.
The information is on the board.
FACTS
The people are looking for information on the sign.
A man is pulling his luggage behind him.
The woman is standing closest to the sign.
The windows are very big.
Some of the people have luggage.
The woman is holding two bags.
The young man is wearing shorts.
1
Z02_TOEIC_5E_Advanced_1441_AnsKey_TOEIC_Advanced 1/5/12 1:03 PM Page 2
INTERPRETATIONS
They are getting ready to board the plane.
They are waiting for their friends to arrive.
The woman is waiting for her husband.
The young man is the woman’s son.
A flight was delayed.
They are not traveling together.
They are looking for information about flights.
Similar Sounds
1. The floor is shiny.
2. They’re standing on the floor.
3. They’re looking at the board.
4. The woman is carrying two bags.
5. The travelers are in front of the information board.
Prepositions
1. The clock is on the information board.
2. A man is walking behind the board.
3. The plant is next to the window.
4. The travelers are standing in front of the board.
5. The woman is holding her bags next to her.
Inaccurate Description
1. The travelers are holding their bags.
2. The man is wearing a vest.
3. The travelers are reading flight information.
Copyright © 2012 by Pearson Education, Inc. Permission granted to reproduce for classroom use.
Different Context
Possible answers:
1. The travelers are checking the flight information.
2. They’re looking at the information board.
3. The woman is reading the sign.
4. The men are wearing glasses.
Photo 2 (page 7)
Language Strategies
Possible answers:
NOUNS
train man woman women platform hats
Connecticut commuters purse shoes coat jacket
raincoat stockings suit hair door window
tracks logo shadow numbers stripe circle
bag day earring heels lady gentleman
people building walkway reflection coat of arms vent
VERBS
stand walk leave arrive depart get off
return follow stop wait drop off visit
carry wear pause rest open step
LOCATIONS
The people are on the platform.
The train is at the station.
The people are walking beside the train.
The train’s logo is to the right of the door.
The hats are on the people’s heads.
The white shoes are on the woman’s feet.
The building is behind the train.
FACTS
The sun is shining.
Copyright © 2012 by Pearson Education, Inc. Permission granted to reproduce for classroom use.
INTERPRETATIONS
The people are commuting to work.
The people live in Connecticut.
The woman is walking ahead of her friends.
The people don’t know each other.
The train is picking up commuters.
There is nobody waiting for the train.
The train is heading north.
Similar Sounds
1. The passengers are getting off the train.
2. The train is on the platform.
3. They’re all carrying bags.
4. They’re getting off the train.
5. The riders are on the platform.
Prepositions
1. The three are walking beside the train.
2. The riders are getting off the train.
3. The passengers are walking on the platform.
4. The woman in the white coat is in front of the others.
5. The woman in the black coat is closest to the door.
Inaccurate Description
1. The train doors are open.
2. The riders are getting off the train.
3. The travelers are ending their journey.
4. The passengers are walking beside the train.
Copyright © 2012 by Pearson Education, Inc. Permission granted to reproduce for classroom use.
Different Context
Possible answers:
1. The women are getting off the train.
2. The passengers are wearing coats.
3. The woman has a bag in her hand.
NOUNS
doctor glasses neck desk
patient eyeglasses hand chair
man pen elbow physician
woman notepad hair paper
telephone folder coat shoulder
phone stethoscope plant notes
VERBS
talk explain write
advise listen hold
ask discuss sit
chat suggest worry
prescribe wear think
LOCATIONS
The phone is on the desk.
The plant is next to the doctor.
The pen is in the doctor’s hand.
The physician is sitting at the desk.
The doctor’s elbows are on the desk.
The stethoscope is around the doctor’s neck.
The doctor is sitting on the chair.
Copyright © 2012 by Pearson Education, Inc. Permission granted to reproduce for classroom use.
FACTS
The doctor is listening to the patient.
The physician is holding his glasses in his hand.
The doctor is in his office.
The notepad is closed.
The man is wearing a white coat.
The woman has long hair.
The doctor isn’t wearing his glasses.
INTERPRETATIONS
The doctor is discussing something with the patient.
The doctor is going to take notes.
The patient is asking the doctor for advice.
The woman is worried about her health.
The physician is going to prescribe some medicine for the woman.
The doctor is thinking about the patient’s problem.
The patient gave the plant to the doctor.
Similar Sounds
1. The doctor is talking with the patient.
2. The desk is between the doctor and the patient.
3. The doctor has a pen in his hand.
4. The woman is facing the doctor.
5. The doctor has a stethoscope.
Prepositions
1. The doctor and the patient are sitting across from each other.
2. The notepad is next to the folder.
3. The phone is on the desk.
4. The plant is next to the doctor.
5. The glasses are in the doctor’s hand.
Inaccurate Description
1. He’s holding his glasses.
2. The doctor has a stethoscope around his neck.
3. The folder on the desk is closed.
4. He’s holding a pen and glasses.
Copyright © 2012 by Pearson Education, Inc. Permission granted to reproduce for classroom use.
Different Context
Possible answers:
1. The doctor has a pen.
2. The doctor is holding his glasses.
3. The doctor has something in his hand.
NOUNS
man watch pillows wall hotel
businessman shirt computer floor room
wrist tie cabinet carpet light
fist pants lamp rug nightstand
hand bed clock time phone
chin bedspread telephone hour
VERBS
look rest turn on
check sleep turn off
plan lie down take a break
think sit read
work lean write
LOCATIONS
The phone is on the nightstand.
The lamp is between the phone and the clock.
The nightstand is next to the bed.
The computer is on the bed.
The man is sitting on the bed.
The pillows are behind the computer.
The man is in front of the nightstand.
Copyright © 2012 by Pearson Education, Inc. Permission granted to reproduce for classroom use.
FACTS
The man is checking the time.
The businessman is looking at his watch.
The computer is open.
The lamp is turned on.
The man is wearing a tie.
There are pillows on the bed.
The cabinet is closed.
INTERPRETATIONS
The man is worried about the time.
The businessman has an appointment.
He’s working in his hotel room.
He’s expecting someone to arrive.
The man is thinking about his work.
He’s waiting for a phone call.
He’s taking a break from his work.
Similar Sounds
1. The man is working in his hotel room.
2. He’s worried about his appointment.
3. He’s looking at the time.
4. There is a clock near the phone.
5. His feet are on the floor.
Prepositions
1. The bed is next to the nightstand.
2. The man is on the bed.
3. The computer is in front of the pillows.
4. The lamp is on the nightstand.
5. The clock is next to the lamp.
Inaccurate Description
1. The computer is on the bed.
2. He’s looking at his watch.
3. The man is sitting on the bed.
4. The businessman is sitting in his hotel room.
Copyright © 2012 by Pearson Education, Inc. Permission granted to reproduce for classroom use.
Different Context
Possible answers:
1. The man is looking at his watch.
2. He’s checking the time.
3. He’s wearing a tie.
NOUNS
man glasses chair day information
woman eyeglasses table paper report
businessman shirt window sunlight colleague
businesswoman coffee cup shade tabletop seat
tie mug blind building reflection
jacket notebook view document
VERBS
talk look drink
discuss listen hold
explain review sit
show ask work
point wear think
LOCATIONS
The mug is on the table.
The blinds are on the window.
The man and woman are sitting at the table.
The man is next to the woman.
The coffee cup is next to the woman.
The document is in front of the man.
The mug is near the edge of the table.
Copyright © 2012 by Pearson Education, Inc. Permission granted to reproduce for classroom use.
FACTS
The man and woman are discussing the documents.
The woman is talking to the man.
The man is listening to the woman.
The blinds on two of the windows are open.
The woman is wearing glasses.
The man is wearing a tie.
The notebook is closed.
INTERPRETATIONS
They’re preparing a report.
The woman is explaining the information to the man.
The woman is drinking coffee.
The man is reviewing the woman’s work.
They’re at their office.
They’re reading a document together.
They’re business colleagues.
Similar Sounds
1. They’re reading the document.
2. They’re meeting in the conference room.
3. The woman is drinking coffee.
4. There is a view of a building in the window.
5. She’s showing the report to the man.
Prepositions
1. The man is in front of the windows.
2. The man and the woman are sitting next to each other.
3. The coffee cup is on the table.
4. The chairs are around the table.
5. They’re sitting at the table.
Inaccurate Description
1. She’s wearing her glasses.
2. He’s looking at the document.
3. The mug is next to the woman.
4. The woman is showing a document to the man.
Copyright © 2012 by Pearson Education, Inc. Permission granted to reproduce for classroom use.
Different Context
Possible answers:
1. The windows have blinds.
2. She is pointing at the paper.
NOUNS
room pillows nightstand hall lights
bedroom picture table door entrance
hotel wall chair painting blanket
lamps floor mat sheets landscape
bed carpet ashtray cover corner
bedspread rug wood mattress
VERBS
stay set check out fluff
sleep hang make spread
rest enter clean tidy up
sit leave organize put
lie check in neaten arrange
LOCATIONS
The pillows are at the head of the beds.
The blankets are on the beds.
The lamps are over the beds.
The chair is next to the table.
The lights are on the wall.
One bed is next to the nightstand.
The ashtray is in the middle of the table.
Copyright © 2012 by Pearson Education, Inc. Permission granted to reproduce for classroom use.
FACTS
The room is vacant.
There are two beds in the room.
The door is closed.
The beds are made.
Each bed has three pillows.
The beds are in a corner of the room.
The beds are against the wall.
INTERPRETATIONS
There are clean sheets on the beds.
The room is ready for guests.
The beds are comfortable.
The guests haven’t checked in yet.
The hotel is expensive.
Housekeeping has just tidied up.
Smoking is allowed in this hotel.
Similar Sounds
1. The pillows are on the bed.
2. The mat is on the table.
3. There is a landscape on the wall.
4. There is a picture near the beds.
5. A carpet lies on the floor.
Prepositions
1. The beds are next to each other.
2. The painting is over one of the beds.
3. There is a mat under the ashtray.
4. The pillows are on the beds.
5. The table is in front of the chair.
Inaccurate Description
1. The beds are made.
2. There are two beds in the room.
3. The lamps are hanging over the beds.
4. The ashtray is on the table.
Copyright © 2012 by Pearson Education, Inc. Permission granted to reproduce for classroom use.
Different Context
Possible answers:
1. The lights are near the beds.
2. There is a painting hanging on the wall.
3. The spreads are on the beds.
VERBS
pour set read
serve put arrange
eat line up look at
drink place sit
stand enjoy fill
LOCATIONS
The cups are on the table.
The teapot is between the cups.
The cups are on saucers.
The books are on the shelf.
The photograph is in the frame.
The picture is between the two groups of books.
The vase is next to a teacup.
Copyright © 2012 by Pearson Education, Inc. Permission granted to reproduce for classroom use.
FACTS
The table is set for tea.
There are two cups on the table.
The cups have decorations on them.
The table is against the wall.
Nobody is sitting at the table.
There are flowers in the vase.
The table has curved legs.
INTERPRETATIONS
Two people are going to enjoy tea together.
The tea is ready to be served.
The cookies are freshly baked.
The cups are empty.
The pot is filled with tea.
This is an expensive café.
The furniture is antique.
Similar Sounds
1. The cups are on the table.
2. Nobody is sitting on the chairs.
3. A picture frame stands on the shelf near the books.
4. The roses smell good.
5. The books stand behind the table.
Prepositions
1. The flowers are in the vase.
2. The saucers are under the cups.
3. The table is between the chairs.
4. The books are next to each other.
5. The picture is on the shelf.
Inaccurate Description
1. The books are closed.
2. The table is against the wall.
3. There is a photo in the frame.
4. The cups are clean.
Copyright © 2012 by Pearson Education, Inc. Permission granted to reproduce for classroom use.
Different Context
Possible answers:
1. The books are on a shelf by the table.
2. The chairs are ready for seating.
NOUNS
car store sky curb windshield
trees building cloud crosswalk apartment
street window palm entrance office
road door leaves plant roof
pavement awning trunk flower row
sidewalk shade bench tire shadow
VERBS
park drive line
shop return wait
buy shine sit
work grow protect
live sell cover
LOCATIONS
The car is parked by the curb.
There are awnings above some of the windows.
The bench is in front of the door.
The bench is between the car and the store.
There are plants growing along the sidewalk.
The car is between two trees.
There are shadows on the sidewalk.
Copyright © 2012 by Pearson Education, Inc. Permission granted to reproduce for classroom use.
FACTS
There are clouds in the sky.
There is only one car in the picture.
The windows are closed.
The car is a convertible.
The bench is empty.
The buildings are two stories high.
It is a sunny day.
INTERPRETATIONS
The stores are closed today.
The driver will return soon.
There are apartments and offices above the stores.
No one is on the street because it is early in the morning.
The beach is nearby.
The weather is warm.
Similar Sounds
1. The car is parked on the street.
2. The car is next to the curb.
3. The buildings have flat roofs.
4. There is a bench by the car.
5. His apartment is above the store.
Prepositions
1. The car is in front of the store.
2. The awnings are over the windows.
3. The tree is in front of the store.
4. The car is next to the sidewalk.
5. The bench is next to the palm tree.
Inaccurate Description
1. There are no people on the street.
2. The doors of the stores are closed.
3. The palm trees are very tall.
4. The car is parked at the side of the road.
5. No one is sitting on the bench.
Copyright © 2012 by Pearson Education, Inc. Permission granted to reproduce for classroom use.
Different Context
Possible answers:
1. There are stores in the building.
2. The car parked near some trees.
3. Trees line the sidewalk.
NOUNS
stove counter slices wood
pot tile pieces brick
lid kitchen pepper handle
soup knife tomato salad
fire cutting board stem reflection
burner cucumber vegetables gas
VERBS
cook heat wash
cut boil peel
slice burn cover
eat turn on chop
prepare turn off light
LOCATIONS
The pot is on the stove.
The vegetables are next to the stove.
The stove is in the kitchen.
The cucumber is on top of the knife.
The tomato is behind the pepper.
The lid is on the pot.
The cutting board is on the counter.
Copyright © 2012 by Pearson Education, Inc. Permission granted to reproduce for classroom use.
FACTS
One burner is lit.
The cucumber is partially sliced.
The stove is turned on.
There are three kinds of vegetables on the cutting board.
The cutting board is made of wood.
The stove has four burners.
The pot is covered.
INTERPRETATIONS
Dinner is being prepared.
Someone is making a salad.
Someone is going to chop the pepper.
There is soup in the pot.
The pot is about to boil.
The vegetables are fresh.
Similar Sounds
1. Someone is cooking soup.
2. The cucumber is cut on the board.
3. A fire is burning on the stove.
4. They haven’t cut the pepper yet.
5. The counter is covered with tiles.
Prepositions
1. The vegetables are on the cutting board.
2. The lid is on the pot.
3. The tomato is behind the pepper.
4. The cutting board is next to the stove.
5. The pot is on the burner.
Inaccurate Description
1. The vegetables are on the cutting board.
2. Someone sliced the cucumber.
3. The counter is made of tiles.
4. The pot is covered.
5. There is one pot on the stove.
Copyright © 2012 by Pearson Education, Inc. Permission granted to reproduce for classroom use.
Different Context
Possible answers:
1. The cook lit the fire on the stove.
2. The handle of the pot is hot.
NOUNS
subway lights bricks floor
train passengers stripe ceiling
station flag metal commuters
platform sign railing transportation
doors numbers seat metro
windows words track railway
VERBS
open ride sit stop
close walk hold on move
leave go slide wait
depart stand rush
arrive operate hang
LOCATIONS
The subway train is in the station.
The train is beside the platform.
The passengers are on the train.
There is a number on the side of the train.
There are metal railings inside the train.
The sign is at the top of the window.
The cutting board is on the counter.
Copyright © 2012 by Pearson Education, Inc. Permission granted to reproduce for classroom use.
FACTS
The train doors are open.
The train isn’t crowded.
There is a man walking along the platform.
The platform is made of tiles.
This is train number 4.
No one is getting on or off the train.
A flag has been painted on the side of the train.
INTERPRETATIONS
The train is ready to depart.
The doors are about to close.
Everyone on the train has a seat.
The train isn’t moving now.
The train is long.
The station is underground.
The man on the platform just got off the train.
Similar Sounds
1. The passengers are sitting in the train.
2. There are windows along the side of the train.
3. There are lights hanging from the ceiling.
4. The platform is in the station.
5. A man is walking on the platform.
Prepositions
1. The flag is next to the door.
2. The lights hang over the platform.
3. A man is walking beside the train.
4. The train is beside the platform.
5. There is a number 4 on the side of the train.
Inaccurate Description
1. The subway train is standing in the station.
2. The train is full.
3. The windows are closed.
4. The doors are open on one side of the train.
Copyright © 2012 by Pearson Education, Inc. Permission granted to reproduce for classroom use.
Different Context
Possible answers:
1. There is a flag by the door of the train.
2. The sign is at the top of the window.
9. (B)
(pages 35–40)
Similar sounds: (A) cove and stove;
(C) ice and slice; (D) cord and board. 21. (B)
10. (A) (A) There are five people looking at the
Similar sounds: (B) plane and train; (C) bag information board but only four of them
and flag; (D) rain and train. are men. (C) A man is carrying his
suitcases but he isn’t opening them. (D)
Exercise 3: Prepositions (pages 35–40) The floor looks very clean and shiny.
11. (A) 22. (D)
(B) The clock is at the top of the board. (C) (A) The train is at the station, but there is
The sign is in front of the windows. (D) no street visible. (B) Some trains have a
The people are standing in front of the first class section, but the passengers are
board. no longer on the train. (C) The front of the
12. (B) train is not visible and there is no
(A) The group is walking away from the conductor in the photo.
door. (C) The woman in white is in front 23. (C)
of the others. (D) The rain is at the station. (A) The doctor is resting his elbow on the
13. (D) desk. (B) The doctor is holding his glasses.
(A) The doctor is sitting across from the (D) The doctor is holding a pair of
patient. (B) The pen is in the doctor’s eyeglasses.
hand. (C) The doctor’s hand is resting on
the folder.
3. (D) 8. (A)
4. (B) 9. (B)
4. (C) 9. (C)
5. (C) 10. (A)
5. (B) 10. (D)
Exercise 6: How (page 48)
Where (page 61)
1. (B) 6. (B)
1. (D) 6. (D)
2. (A) 7. (C)
2. (C) 7. (B)
3. (C) 8. (A)
3. (A) 8. (B)
4. (C) 9. (C)
4. (B) 9. (D)
5. (A) 10. (B)
5. (C) 10. (A)
Exercise 7: Auxiliaries (page 49)
Why (page 63)
1. (C) 6. (A)
1. (D) 6. (A)
2. (B) 7. (B)
2. (C) 7. (C)
3. (B) 8. (B)
3. (C) 8. (A)
4. (B) 9. (B)
4. (B) 9. (C)
5. (A) 10. (C)
5. (B) 10. (D)
ANSWER KEY
LISTENING COMPREHENSION REVIEW
PART 1 (PAGES 92–97) 6. (C) Two housekeepers are making the bed
in a hotel room. Choice (A) uses words
1. (A) The chef is displaying his food on a
associated with hotels: clerks, greeting,
banquet table. Choice (B) uses a word
guests. Choice (B) uses the associated
associated with food: produce. Choice (C)
word sheets, which the housekeepers are
confuses the man’s occupation and
putting on the bed. Choice (D) uses the
activity; he is a chef presenting food, not a
similar word service, but room service
waiter serving it. Choice (D) uses an
implies delivering food, not cleaning
associated word, vegetables.
the room.
2. (C) The highway has six lanes. Choice (A)
7. (B) There are some suitcases going around
associates service station and gas with the
the conveyor belt that carries luggage
vehicles in the picture. Choice (B) is
from the plane to the baggage claim area.
incorrect because the cars are moving, not
Choice (A) uses the associated words
stopped at a rest stop. Choice (D) confuses
travelers, packing, and bags, but the words
the similar-sounding words rodeo and
are in a different context. Choice (C)
road.
confuses the similar-sounding words suits
3. (D) Pedestrians are crossing in the middle and suitcases, conveyor and chair. Choice
of the street. Choice (A) uses the correct (D) uses words associated with travel,
word pedestrians, but they are not waiting passengers and plane, but the statement
for a traffic light. Choice (B) confuses the does not match the picture.
location of the cars; they are moving, not
8. (A) A hotel doorman or bellhop is holding
parked. Choice (C) confuses the similar-
the door of a taxi open for an arriving
sounding words carts and cars.
guest. Choice (B) uses the similar-
Copyright © 2012 by Pearson Education, Inc. Permission granted to reproduce for classroom use.
4. (A) The display of dials and monitors is sounding word tax for taxi. Choices (C)
probably some massive control panel and (D) misidentify the occupation and
which the man is monitoring. Choice (B) the activity.
confuses watching the monitors with
9. (B) The helicopter is landing. Choice (A)
watching television. Choice (C) is incorrect
confuses the similar-sounding words cup
because the man is standing inside a
and helicopter. Choice (C) confuses the
building, not out in the rain. Choice (D)
blades of grass are blowing and the blades of
confuses the similar-sounding words
the helicopter are moving, which may cause
patrol and control.
the grass to blow, but there is no grass in
5. (C) A speaker is standing and gesturing in the picture. Choice (D) tries to confuse by
front of the chart on the wall. In choice (A) using the word sheep for ship.
the woman might be a trainer, but she is
10. (D) The technicians are conducting tests
not writing on a board. Choice (B) is
on a satellite before it is sent into space.
incorrect because the woman is wearing a
Choices (A), (B), and (C) use words in a
suit but not a tie. Choice (D) is incorrect
different context. The people in the photo
because three men and one woman are
are not musicians, doctors, or astronauts,
sitting at the table, not three women.
and they are not performing any of the
listed activities.
PART 2 (PAGE 98) 20. (A) I like the way it sounds answers why by
providing an explanation. Choice (B)
11. (A) A person who doesn’t feel well should
repeats the word keep but does not answer
see a doctor. Choice (B) repeats the word
the question. Choice (C) confuses the
well in a different context. Choice (C)
similar-sounding words ringing and
repeats the word feel in a different context.
singing.
12. (B) Your mother answers who. Choice (A)
21. (C) I do, thank you is a polite response to
associates being out of the office with
the question. Choice (A) is not a proper
missing a person. Choice (C) associates
response. Choice (B) repeats the word car
putting the cat out with being out of
but does not answer the question.
the office.
22. (A) A storm warning answers what. Choice
13. (C) It gets in at three answers when. Choice
(B) is not a logical response. Choice (C)
(A) answers how the plane might land.
confuses the similar-sounding words
Choice (B) associates Hawaii with
weather and whether.
Honolulu.
23. (B) At eight answers when. Choice (A)
14. (C) A camera shop answers where. Choice
confuses the similar-sounding words new
(A) confuses the similar-sounding words
and news and associates television with
envelope and developed. Choice (B)
news. Choice (C) associates programs
associates film (movie) with film
with news.
(photographs).
24. (A) In the hospital answers where. Choice
15. (A) That will be easy to arrange means that
(B) answers when. Choice (C) confuses the
the speaker agrees with having the
similar-sounding words hospitable and
meeting here. Choice (B) confuses the
hospital.
similar-sounding words met and meeting.
Choice (C) confuses the similar-sounding 25. (B) It’s hand-knit provides a reason for the
words eat and meet. sweater’s high cost. Choice (A) confuses
the similar-sounding words coastal and
16. (C) If a restaurant turns out to be too
cost and associates hot with sweater. Choice
expensive, it’s logical to choose another
(C) associates heat with sweater.
place to eat in the future. Choice (A)
confuses rest with the similar-sounding 26. (C) The first speaker doesn’t like the cold
Copyright © 2012 by Pearson Education, Inc. Permission granted to reproduce for classroom use.
word restaurant. Choice (B) associates weather, and the second agrees by saying
expense with the word expensive. me neither. Choice (A) uses the word stand
in a different context. Choice (B) confuses
17. (A) Pick up some stamps is a logical request
together with the similar-sounding word
to make of someone going to the post
weather.
office. Choice (B) repeats the word lunch.
Choice (C) uses the word post in a 27. (B) Soup answers what. Choice (A)
different context. associates hungry with lunch. Choice (C)
answers would you like to go to lunch.
18. (B) Two o’clock is a specific time that
answers when. Choice (A) confuses the 28. (C) When I have enough money answers
similar-sounding phrases two clocks and when. Choice (A) associates house with
two o’clock. Choice (C) confuses the housekeeper. Choice (B) associates kitchen
similar-sounding phrases pick fruit and with house.
pick you up. 29. (A) Why bother suggests that there is no
19. (C) At the airport answers where. Choice need to get the shoes shined. Choice (B)
(A) associates package with luggage. Choice confuses similar-sounding words shoes,
(B) confuses the object of the question. shined with news, time. Choice (C) repeats
the word shoes.
30. (B) I’ve seen all the films is a polite refusal 40. (C) It needs paving answers why the road is
to the invitation. Choice (A) begins with a bumpy. Choice (A) confuses the words
proper response but does not answer the rode and road. Choice (B) associates trucks
question. Choice (C) associates director with road.
with movies.
PART 3 (PAGES 99–101)
31. (C) There was a big sale, so the second
speaker went shopping. Choice (A) 41. (B) The woman says today is Monday.
associates jazz with music. Choice (B) Choice (A) confuses Monday with the
confuses sailing with the similar-sounding similar-sounding word Sunday. Choice (C)
word sale. confuses Tuesday with the similar-
sounding word today. Choice (D) is when
32. (B) The secretary answers who. Choice (A)
the speakers started some work.
associates paper and newspaper. Choice (C)
confuses the similar-sounding words knew 42. (B) The man has a dentist appointment
and newspaper. next week. Choice (A) is what he will do
tomorrow. Choice (C) is something the
33. (C) On Tuesday answers when. Choice (A)
woman suggests, but the man says he
does not answer the question. Choice (B)
doesn’t need this. Choice (D) uses the
confuses when the delegates were selected
word appoint in a different context.
and when the delegates will arrive.
43. (C) The woman says that they will finish
34. (A) At the department store answers where.
writing the conference report that they
Choice (B) repeats radio. Choice (C)
started last Friday night. Choice (A)
associates heard with radio.
confuses fight with the similar-sounding
35. (A) My favorite camera just broke provides a word night. Choice (B) confuses walk with
reason for the speaker’s unhappiness. the similar-sounding word work. Choice
Choice (B) associates funny with happy by (D) repeats the word conference, but we
confusing the similar-sounding words can assume they have already attended
happy and unhappy. Choice (C) confuses the conference, since they are writing a
the similar-sounding words happen and report about it.
unhappy.
44. (A) The man mentions the blue tie. Choice
36. (B) The largest answers what size. Choice (B) confuses white with the similar-
Copyright © 2012 by Pearson Education, Inc. Permission granted to reproduce for classroom use.
(A) associates strong with size. Choice (C) sounding word like. Choice (C) confuses
associates trunk with luggage and half gold with the similar-sounding word hold.
with size. Choice (D) confuses pink with the similar-
37. (C) I do answers who. Choice (A) confuses sounding word think.
exchanging money and exchanging a 45. (C) The man says it really goes well with my
purchase. Choice (B) confuses the similar- suit. Choice (A) uses the word suit in a
sounding words changed and exchange. different context. Choice (B) is not
38. (B) No, it hasn’t arrived yet provides a mentioned. Choice (D) is not true because
logical response to a yes/no question. the man says he never pays that much for
Choice (A) confuses the similar-sounding a tie.
words reception and receive. Choice (C) 46. (C) The tie costs $34. Choices (A), (B), and
associates sign with letter. (D) all sound similar to the correct answer.
39. (A) In the top drawer answers where. Choice 47. (A) The man says let’s enjoy this morning
(B) confuses the similar-sounding words sunshine. Choices (B) and (C) are
directory and dictionary. Choice (C) mentioned by the woman when she talks
confuses the similar-sounding words about the future weather. Choice (D)
dictation and dictionary. confuses at night with tonight.
48. (D) The man says he will stay home. 57. (B) The man says he will be leaving in two
Choices (A) and (C) confuse work and walk weeks. Choice (A) is not mentioned.
with the similar-sounding word week. Choice (C) is the length of his next trip.
Choice (B) is mentioned by the woman as Choice (D) is not correct.
something she has already done.
58. (C) The man asks the woman to forward
49. (D) The man says that clear skies are his mail. Choice (A) associates letter with
reported for tomorrow. Choice (A) is how mail. Choice (B) confuses e-mail with mail.
the weather will be this afternoon. Choice Choice (D) confuses meal with the similar-
(B) confuses freezing with the similar- sounding word mail.
sounding word breeze. Choice (C) is how
59. (A) The woman, who says she normally
the weather may be this evening.
does administrative work, is the one who
50. (C) The woman says I’ve been in a meeting is falling behind. Choices (B) and (C) are
all day. Choices (A) and (B) confuse eating other people mentioned in the
and reading with the similar-sounding conversation. Choice (D) associates
word meeting. Choice (D) is what the man publishers with book, but the word book is
was doing. used in this context to mean make
51. (B) The man was waiting for a flight, so he reservations.
must have been at the airport. Choice (A) 60. (A) She has to book lodging, meetings,
is where he wanted to go. Choice (C) is lunches, and trips. Choices (B), (C), and
what he did later. Choice (D) is not (D) are all associated with the activities
mentioned. she has to book for the visitors.
52. (C) At the travel agency, the man was told 61. (D) The visitors will leave on Thursday
that he will probably have to wait until night. Choice (A) confuses afternoon with
Sunday. Choice (A) confuses tonight with the similar-sounding word soon. Choice
the similar-sounding word flight. (B) confuses tonight with night. Choice (C)
Choice (B) is the woman’s guess. confuses Tuesday with the similar-
Choice (D) confuses Monday with the sounding word Thursday.
similar-sounding word Sunday.
62. (B) The order is still at the factory; it
53. (A) The woman says This hotel would be a hasn’t been shipped yet. Choice (A) is
Copyright © 2012 by Pearson Education, Inc. Permission granted to reproduce for classroom use.
good place for our next convention. Choice where the order will go when there is
(B) is where the convention was last year. enough room. Choice (C) confuses store
Choices (C) and (D) are places that are with storage. Choice (D) confuses ship with
near the hotel. shipment.
54. (B) The man says we still have six months to 63. (B) The woman wants the man to hold the
plan. Choice (A) confuses two with to. order for 14 days, or two weeks. Choices
Choice (C) confuses nine with the similar- (A), (C), and (D) are numbers that sound
sounding word time. Choice (D) confuses similar to 14.
in a year with last year, which is when the
64. (D) The man says that the price is seventy-
last convention was held.
five fifty a week. Choices (A), (B), and (C)
55. (C) The woman says that they might not sound similar to the correct answer.
even have enough money in the budget for it.
65. (A) The woman says that she bought a
Choices (A), (B), and (D) are things that
desk. Choice (B) confuses chair with the
she likes about the place.
similar-sounding word where. Choice (C)
56. (D) The man says that he has to buy a confuses lamps with the similar-sounding
bigger suitcase. Choice (A) confuses suit word cramped. Choice (D) is mentioned by
with suitcase. Choice (B) confuses tie with the woman, but it isn’t what she buys.
the similar-sounding word tired. Choice
(C) associates airline tickets with trip.
66. (A) The woman bought the desk at the 74. (A) The remote control transmitter
mall last Saturday. Choices (B), (C), and engages the car’s ignition. Choice (B)
(D) sound similar to the correct answer. confuses home furnace with prewarming
from your home. Choice (C) associates
67. (D) The woman says that the other side of
automatic oven with automatically
the room is a convenient place for the desk
prewarmed. Choice (D) confuses the
because the computer equipment is there.
similar-sounding words cooler and
Choice (A) confuses hall with the similar-
precooled.
sounding words mall and all. Choice (B)
confuses closet with the similar-sounding 75. (A) The transmitter shuts off after 15
word closed. Choice (C) is the man’s minutes. Choice (B) is not mentioned.
suggestion for a place to put the desk. Choice (C) confuses 15 hours and 15
minutes. Choice (D) confuses 20 hours and
68. (C) Part-time employees get two and a
20 degrees.
half days of vacation time. Choice (A) are
the people who get a week of vacation. 76. (D) The transmitter is available at
Choice (B) confuses employer with the Farenham’s auto supply stores. Choice (A)
similar-sounding word employees. Choice confuses the similar-sounding Fahrenheit
(D) is incorrect because the benefits are and Farenham’s. Choice (B) confuses the
different for part-time and full-time similar-sounding words Aaron and
employees. Fahrenheit. Choice (C) is not mentioned.
69. (B) The woman says They also get a day of 77. (C) Universal makes men’s suits in the
sick leave each year. Choices (A), (C), and medium to high price range. Choice (A)
(D) sound similar to two and a half, which confuses the similar-sounding phrases
is the number of vacation days they get. In boys’ pants and Paris, France. Choice (B)
addition, choice (D) is the number of confuses factory parts and factory in Paris.
vacation days they get. Choice (D) associates French and France.
70. (B) The man says That’s not very much. 78. (B) The original Hong Kong factory will
Choice (A) repeats the word much. Choice be closed down. Choice (A) confuses the
(C) repeats the word unpaid. Choice (D) is similar-sounding words Lawrenceville and
contradicted by the correct answer. Lawrence. Choice (C) confuses the similar-
Copyright © 2012 by Pearson Education, Inc. Permission granted to reproduce for classroom use.
81. (B) The activities have been planned 87. (C) Orphans are children without parents.
according to the weather. Choice (A) is not Choice (A) will match the donations.
mentioned. Choice (C) is not mentioned as Choice (B) is making the announcement.
an influential factor. Choice (D) is not Choice (D) confuses the similar sounds in
mentioned. the words elephant and orphan.
82. (C) Shipboard activities are arranged for 88. (A) Station XYZ presented the recording
the duration of the three-day cruise. as a public service announcement. Choice
Choice (A) confuses three hours and three (B) is where the money will be donated.
days. Choice (B) confuses the similar- Choice (C) confuses Public Service
sounding words thirteen and third. Choice Association and public service
(D) confuses the similar-sounding words announcement. Choice (D) will match the
thirty and third. donations.
83. (D) The electric company made the 89. (D) The tour is taking place at the Natural
recording. Choice (A) confuses phone History Museum. Choice (A) confuses
company and the phones being engaged. recording studio and prerecorded tour.
Choice (B) confuses musician with hearing Choice (B) associates army with guards.
a recording and recorded music. Choice (C) Choice (C) associates prison with guards.
associates doctor with operation and
90. (B) A description and an explanation of
patients by confusing the similar-sounding
each exhibit have been recorded. Choice
words operation and operator, and patients
(A) confuses the similar-sounding words
and patience.
activities and activated. Choices (C) and (D)
84. (A) All phones are engaged at the are not mentioned.
moment. Choice (B) is incorrect or else the
91. (D) Museum guards are posted along the
caller probably would not have heard the
way. Choice (A) confuses the similar-
recording. Choice (C) confuses the words
sounding words poster and posted. Choices
record (noun) and record (verb). Choice (D)
(B) and (C) are not mentioned.
associates operation and patients with
doctor by confusing the similar-sounding 92. (B) A travel alert has been posted because
words operation and operator, and patients roads are expected to freeze. Choice (A)
and patience. associates heavy commuter traffic with
Copyright © 2012 by Pearson Education, Inc. Permission granted to reproduce for classroom use.
95. (D) The announcement is about wireless 99. (B) Marcel mentions getting the software to
Internet access at the mall. Choices (A) work and being on a really difficult
and (B) are associated with a mall but are assignment, so he is probably working
not the correct answer. Choice (C) with a client. Choice (A) confuses the
repeats the word wireless. meeting he is calling to reschedule.
96. (D) The computers will be placed near Choice (C) is incorrect because Marcel
the entrance. Choice (A) confuses hall never mentions the word vacation. Choice
with the similar-sounding word mall. (D) is associated with accounting software.
Choices (B) and (C) are other places in 100. (A) Monday is the day that Marcel says
the mall where Internet access will be he can’t meet. Choices (B), (C), and (D)
available. are the days that Marcel is available to
97. (A) People who attend the inauguration meet.
celebration will get a coupon for a free
cup of coffee. Choice (B) mentions
spouses, but they are not connected with
getting free coffee. Choices (C) and (D)
are people who will be interested in
using the Internet access service.
98. (C) The message is about rescheduling a
meeting, so it must be for a co-worker.
Choice (A) is someone Marcel might call
while he is away on a trip but not to
schedule a meeting. Choice (B) is
associated with scheduling an
appointment. Choice (D) associates being
away on a trip with a vacation.
Copyright © 2012 by Pearson Education, Inc. Permission granted to reproduce for classroom use.
ANSWER KEY
READING PRACTICE
PART 5: INCOMPLETE SENTENCES Verbs: Two-Word (page 122)
Prepositions (page 107) 1. (B) 6. (C)
1. (C) 6. (D) 2. (A) 7. (B)
2. (D) 7. (B) 3. (D) 8. (B)
3. (D) 8. (A) 4. (C) 9. (D)
4. (C) 9. (D) 5. (B) 10. (A)
5. (A) 10. (B) Word Families (page 124)
Coordinate Conjunctions (page 109) 1. (C) 6. (B)
1. (C) 6. (A) 2. (D) 7. (D)
2. (D) 7. (D) 3. (C) 8. (A)
3. (A) 8. (D) 4. (A) 9. (C)
4. (B) 9. (C) 5. (D) 10. (C)
5. (B) 10. (A) Similar Words (page 126)
Subordinate Conjunctions (page 111) 1. (C) 6. (C)
1. (C) 6. (A) 2. (B) 7. (B)
2. (C) 7. (B) 3. (A) 8. (D)
3. (B) 8. (C) 4. (D) 9. (D)
4. (D) 9. (D) 5. (A) 10. (C)
5. (A) 10. (A) Strategy Practice (page 129)
Transition Words (page 113) 1. (C) 11. (B)
1. (D) 6. (B) 2. (B) 12. (C)
2. (A) 7. (C) 3. (A) 13. (D)
3. (C) 8. (A) 4. (A) 14. (A)
Copyright © 2012 by Pearson Education, Inc. Permission granted to reproduce for classroom use.
2. D 7. D Language Strategies
3. D 8. D 1. (B) 6. (C)
4. D 9. V 2. (A) 7. (C)
5. D 10. I 3. (B) 8. (B)
Language Strategies 4. (A) 9. (B)
1. (D) 6. (A) 5. (A) 10. (C)
2. (C) 7. (C)
3. (B) 8. (A) Double Passages (page 176)
4. (B) 9. (D)
Test Strategies
5. (D) 10. (D)
1. I 9. D
2. D 10. D*
Forms and Tables (page 162) 3. D* 11. D
4. D* 12. I
Test Strategies
5. D 13. D
1. M 6. I
6. I 14. D*
2. D 7. D
7. D 15. I*
3. D 8. D
8. D*
4. D 9. D
5. D
ANSWER KEY
READING REVIEW
PART 5 (PAGES 190–193) 108. (C) Causative had requires the simple
form of the following verb file when the
101. (B) Although is a subordinate conjunction
subject does the action. Choice (A) is the
meaning in spite of. Choice (A) is a
infinitive form. Choice (B) is the present
subordinate conjunction that means as a
continuous tense. Choice (D) is the past
result of. Choice (C) is a transition word.
tense.
Choice (D) is a coordinate conjunction.
109. (A) From is a preposition meaning, in this
102. (A) Aren’t is a present tense verb used
case, as a result of. Choice (B) is a
with a real condition. Choice (B) is the
preposition indicating placement. Choice
future tense. Choice (C) is the negative
(C) is a preposition used with location.
future tense. Choice (D) is a form of have,
Choice (D) is a preposition indicating
not of be.
direction.
103. (B) Never is an adverb of indefinite
110. (C) In addition is a transition phrase
frequency, which comes after the
meaning also. Choice (A) is a transition
auxiliary have. Choice (A) has no
phrase that indicates a following
auxiliary. Choices (C) and (D) have
example. Choice (B) is a transition phrase
misplaced indefinite adverbs.
that indicates an alternative. Choice (D)
104. (D) Except means excluding or aside from. is a transition word that indicates an
In this sentence it tells us that Mr. Kim unexpected result.
was the only person who did not attend
111. (C) From is a preposition used with
the meeting. Choices (A), (B), and (C)
origin or source. Choice (A) is a
look similar to the correct answer but
preposition of manner. Choice (B) is a
have meanings which don’t fit the
preposition meaning accompanied. Choice
Copyright © 2012 by Pearson Education, Inc. Permission granted to reproduce for classroom use.
115. (B) To is a preposition meaning toward. 124. (B) Since is a subordinate conjunction
Choice (A) is a preposition indicating meaning because. Choice (A) is a
origin. Choice (C) is a preposition subordinate conjunction meaning
indicating purpose or in place of. Choice unexpected. Choice (C) is a transition
(D) is a preposition used with location. word suggesting an alternative. Choice (D)
116. (B) Next is a transition word that is used is a subordinate conjunction meaning if.
in sequence with first. Choice (A) is a 125. (C) And is a coordinate conjunction that
transition word meaning additionally. is often paired with both. Choices (A),
Choice (C) is a transition phrase (B), and (D) are conjunctions that are not
suggesting an alternative. Choice (D) is a paired with both.
transition word used with results.
126. (B) Causative let requires the simple form
117. (D) Causative wanted requires the past of the following verb work when the
participle of the following verb liquidated subject does the action. Choice (A) is the
when the subject does not do the action. infinitive. Choice (C) is the present
Choice (A) is the infinitive. Choice (B) is participle. Choice (D) is the future tense.
the future tense. Choice (C) is the simple
127. (D) Gave it up means quit. Choice (A) is
form of the verb.
not logical. Choice (B) means distributed.
118. (B) With a real condition, use the present Choice (C) means made it a gift to someone.
tense think. Choice (A) is the past tense.
128. (C) A real condition requires the present
Choice (C) is the past perfect tense.
tense needs in the if-clause. Choice (A) is
Choice (D) is the past perfect continuous
the future tense. Choice (B) is the simple
tense.
form. Choice (D) is the past tense.
119. (C) Until recently means the past up to the
129. (A) Previous year requires the past tense
near present. Use the present perfect have
was. Choice (B) is the present perfect.
been. Choice (A) is the present tense.
Choice (C) is the present tense. Choice
Choice (B) is the future tense. Choice (D)
(D) is the future tense.
is the past continuous tense.
130. (B) Interest is a noun forming part of the
120. (C) Has always been is the present perfect
noun phrase interest rate. Choice (A) is a
tense, indicating an action started in the
present participle. Choice (C) is a past
Copyright © 2012 by Pearson Education, Inc. Permission granted to reproduce for classroom use.
134. (A) Decrease means reduce or make smaller 143. (B) In billing, balance refers to the money
in number. Choices (B) and (C) are similar still owed. Choices (A), (C), and (D)
in meaning: they mean go down but not cannot be correctly used in this context.
make something go down. Choice (D) is not 144. (C) Has been is a present perfect verb,
a verb. indicating that the action started in the
135. (C) The indefinite frequency adverb past, when the bakery opened, and
always can be placed between the continues up to the present. Choice (A) is
auxiliary and main verb. Choice (A) is an present tense. Choice (B) is future.
incorrectly placed adverb. Choice (B) Choice (D) is a present participle.
refers to the past. Choice (D) does not 145. (D) A gerund is required following the
indicate frequency. preposition in. Choice (A) is the base
136. (B) Position means job. Choice (A) is form. Choice (B) is simple past tense.
related in meaning but is a verb, not a Choice (C) is an infinitive.
noun. Choices (C) and (D) mean 146. (D) The writers want the reader to call a
profession, which would refer to Ms. third person, Alan Hunter, or him. Choice
Chang’s skills and experience but not to (A) is a plural form. Choices (B) and (C)
any particular job she may be hired to would refer to the writers of the e-mail,
do. but they aren’t the ones who want to
137. (D) Unreal conditions in the past require receive a call.
a past participle in the result clause, 147. (B) The noun form is required here.
would have increased. Choices (A) and (C) Choices (A) and (C) are adjectives.
are future forms. Choice (B) is the Choice (D) is a verb.
present tense.
148. (A) Opening night is a specific date, so the
138. (A) Therefore is a transition word that is preposition on is used. Choice (B) at night
used to show a result. Choice (B) refers to a time of day. Choice (C) means
indicates a contrast. Choice (C) indicates before. Choice (D) means starting from.
a restatement. Choice (D) indicates an
additional idea. 149. (B) Rarely means almost never, and it is
clear from the story description that this
139. (D) Yet is a transition word that is used to character spends very little time with his
Copyright © 2012 by Pearson Education, Inc. Permission granted to reproduce for classroom use.
show an unexpected result. Choice (A) family. Choices (A), (C), and (D) all
indicates an additional idea. Choice (B) is imply that the character spends more
used in a sequence. Choice (C) is used to time with his family.
show a bad result.
150. (B) Deeply is an adverb used to modify
140. (C) Operating is a present participle the verb regret. Choice (A) is an adjective.
describing the kind of costs. Choice (A) is Choice (C) is a noun. Choice (D) is a
a noun referring to a person. Choice (B) verb.
is a noun referring to a thing. Choice (D)
is a past participle. 151. (A) The past participle had completes the
present perfect verb have had. Choice (B)
PART 6 (PAGES 194–197) is a present participle. Choices (C) and
(D) are present tense verbs.
141. (B) The expression due to means
because of. Choices (A), (C), and (D) are 152. (B) The order was lost, and then it was
prepositions that cannot be correctly found. Choices (A), (B), and (C) do not fit
used with this expression. the context.
142. (C) The check was written by Heather
Taylor, the person to whom the letter is
addressed, so your is the correct
possessive form. Choices (A), (B), and
(D) are incorrect.
168. (A) The increase in male employees in 176. (C) Seasoned globetrotters are experienced
Region I increased by only one. Choice travelers. Choice (A) is not mentioned.
(B) is incorrect because the number of Choice (B) confuses the Harlem
female employees increased by only Globetrotters and seasoned globetrotters.
three, while Region E has the most Choice (D) associates international with
employees overall. Choice (C) is travel and seasoned with gourmets.
incorrect because Region E has the most
177. (B) If travelers change their money before
employees overall. Choice (D) is
they leave, they will not have to wait in
incorrect because Region A has the
line for foreign currency when they
fewest employees overall.
arrive in a foreign country. Choices (A)
169. (C) Region E had the greatest increase in and (C) are things that travelers will still
both female and male employees. have to do. Choice (D) confuses watch the
Choices (A), (B), and (D) all had smaller stock market and hedge against market
increases in employees than Region E. fluctuations.
170. (A) Salary and benefits are not given in 178. (B) Travelers can be confused with the
the ad. Choices (B), (C), and (D) are all various denominations of money. Choice
mentioned. (A) is not logical. Choice (C) is not
171. (D) The ad calls for proven management mentioned. Choice (D) is incorrect
ability in the field of cellular mobile radio because banks, not travelers, apply
and paging. Choice (A) associates foreign exchange commissions.
publications with reporting. Choice (B) 179. (A) The office is centrally located
associates law with administration. downtown. Choice (B) is not mentioned.
Choice (C) is not logical. Choice (C) confuses deluxe suburb and
172. (C) The letter recommends Mr. Fechter deluxe executive office space. Choice (D)
for the position of chief accountant. associates corporate with executive.
Choices (A), (B), and (D) are not logical. 180. (D) Shower facilities are available, but
173. (D) Mr. Fetcher handled new not a health club. Choice (A) is one of the
responsibilities commendably; he responsibilities of a receptionist. Choice
handled them in an admirable way. (B) is a conference room. Choice (C) is
Copyright © 2012 by Pearson Education, Inc. Permission granted to reproduce for classroom use.
Choices (A), (B), and (C) do not fit the the same as a deluxe office.
context. 181. (C) The information in the notice helps
174. (A) Mr. Fechter is currently employed at people decide whether they are eligible
Duchampion Co., Inc., which is unable to for a flu shot. Choices (A) and (B)
promote him at the present time. Choice are related to the topic of the notice
(B) is not mentioned. Choice (C) is but are not mentioned. Choice (D) is
contradicted by Mr. Fechter being a associated with manufacturers, but the
certified public accountant (CPA). Choice manufacturing process is not mentioned.
(D) is where he is applying for a position. 182. (B) According to the notice, there is a
175. (B) Mr. Fechter has experience as an national shortage of flu vaccine this year,
accountant but not as a chief accountant. which is why only certain people are
Choice (A) is not mentioned. Choice (C) eligible to receive it. Choice (A) is
is contradicted by his CPA title. Choice incorrect because it is fewer people in
(D) is contradicted by the positive general, not fewer doctors who are getting
references to his ability. vaccinated. Choice (C) is incorrect
because although nurses are mentioned,
their work availability is not. Choice (D)
is a possible result of the shortage of flu
vaccine, but it is not mentioned.
183. (B) As a health care worker, a doctor falls 189. (B) This web page is the one mentioned
into the category of people eligible for flu in the e-mail: On the homepage you will
shots, but no mention is made of doctors find a private link for shareholders.
getting special treatment when they go to Choice (A) is how people can submit
get vaccinated. We can assume they have biographies. Choices (C) and (D) are
to wait their turn like everyone else. related to the topic but are not
Choice (A) is incorrect because all health mentioned.
care workers are eligible for shots.
190. (B) The web page offers the information
Choice (C) is not mentioned and is not
that A new year brings a new vice president.
likely. Choice (D) contradicts the correct
Choices (A), (C), and (D) are related to
answer.
the topic but are not mentioned.
184. (C) It is pregnant women, or mothers-to-
191. (A) The bill shows a charge of $0 for
be, not fathers-to-be, who will be eligible
long-distance calls. In addition, from the
for flu shots. Unless the father-to-be fits
letter we understand that Katie Miller
any of the criteria noted, he will not be
used to use this company for her long-
eligible for the flu shots. Choice (A) will
distance calls, but that she no longer
be eligible because flu shots will be
does. Choice (B) is incorrect because
offered to all school staff members.
although she is charged for an answering
Choice (B) will be eligible as a health care
service, she wouldn’t be charged by the
worker. Choice (D) will be eligible
phone company for use of an answering
because of being under 12 months of age.
machine. Choice (C) is not true; she used
185. (A) Anyone who brings a letter from a her home phone line and was charged for
doctor can get a flu shot at Apple Square it. Choice (D) refers to a type of long-
Mall. Choice (B) is wrong because nurses distance service offered by the company,
will accept walk-ins. Choice (C) is wrong which Miller didn’t use.
because students must get their flu shots
192. (C) Miller was charged $89.75 and sent
at their schools. Choice (D) is confused
in a payment of $100. Choice (A) is
with the fact that the flu shots are being
incorrect because her payment was
given at the mall, but this condition is
received on September 19, well before
never mentioned.
the September 28 due date. Choice (B)
Copyright © 2012 by Pearson Education, Inc. Permission granted to reproduce for classroom use.
186. (A) The e-mail message says we ask that is the opposite of the correct answer.
you also take a moment to visit our new Choice (D) is the amount she was
website. Choices (B), (C), and (D) are not charged.
mentioned.
193. (B) Miller stopped using this company’s
187. (A) The writer of the e-mail, who is the long-distance service, and the company
CEO, says Others, myself included, will wants her to resume using it. Choice (A)
take a vacation somewhere warm and is incorrect because Miller was not
relaxing. Choice (B) is incorrect because charged anything for long distance.
the plant will be shut down. Choice (C) is Choices (C) and (D) confuse details
not mentioned. Choice (D) is mentioned related to phone service.
as something that some people will do
194. (B) To hear her phone messages, or voice
but not the CEO.
mail, Miller has to dial her own phone
188. (C) The website gives the information number. Choice (A) is Miller’s client
that January was the last edition of the number, not her telephone number.
newsletter. Choices (A), (B), and (D) are Choice (C) is the number to call to sign
things that will continue to be available. up for long distance services. Choice (D)
is the number for billing inquiries.
195. (C) Miller is still a customer with this 199. (B) The information on the web page says
company. She uses it for local calls. No amount is too small. Choices (A) and
Choices (A), (B), and (D) are contradicted (C) are incorrect because last year the
by the correct answer. foundation used funds to build a
196. (D) The web page explains how to make computer lab. Choice (D) is the amount
a donation. Choices (A), (B), and (C) are of money raised last year, but there is no
related to the topic but are not the correct indication that this same amount is
answer. raised every year.
197. (B) The e-mail receipt acknowledges an 200. (B) The receipt was sent as proof of your
online donation. Choices (A), (C), and (D) donation when you file your taxes. Choices
are possible ways to donate, but (A), (C), and (D) are plausible reasons
donations made in these ways receive a but are not the correct answer.
receipt by regular mail.
198. (A) Everyone who makes an online
donation above this amount gets a
receipt. Choices (B), (C), and (D) are
plausible reasons but are not the correct
answer.
Copyright © 2012 by Pearson Education, Inc. Permission granted to reproduce for classroom use.
ANSWER KEY
PRACTICE TEST ONE
PART 1 (PAGES 220–225) 6. (C) A farmer is working in a rice paddy,
tending his rice crop. Choice (A)
1. (C) An uncompleted bridge crosses a
mentions the water, but the worker is
river—it is still under construction.
standing in it, not drinking it. Choice (B)
Choice (A) mentions the river and the
is incorrect because the man is growing
idea of crossing it, but there are no cars
rice, not cooking it. Choice (D) associates
in the picture and cars couldn’t cross the
lake with the water in the rice paddy.
incomplete bridge. Choice (B) mentions
the bridge and associates ships with the 7. (A) The factory workers are working in a
river, but there are no ships in the row, one next to the other, loading boxes.
picture. Choice (D) mentions the hills, Choice (B) associates packages with boxes,
but they are covered with grass, not but the workers are not mailing them.
snow. Choice (C) confuses similar-sounding
words dancing and standing. Choice (D)
2. (B) The food on the table is in bowls.
associates presents with boxes.
Choice (A) confuses the similar words
bowling (a game) and bowls (containers 8. (C) The people staring intently at a
for food). Choice (C) associates pottery monitor are working in an office. Choice
with bowls. Choice (D) associates dish (A) confuses the team in the field and the
with bowl. team of workers in the office. Choice (B) is
incorrect because the employees are
3. (D) A man is signing a paper or
working, not having lunch. Choice (D)
document. Choice (A) is incorrect
confuses the similar sounds of off the ice
because the man is signing the
and office.
document, not typing it. Choice (B)
9. (A) Two pilots are seated in an airplane
Copyright © 2012 by Pearson Education, Inc. Permission granted to reproduce for classroom use.
PART 2 (PAGE 226) 20. (A) By 5:00 answers when. Choice (B)
associates play with theater. Choice (C)
11. (C) The human resources office answers
associates actors with theater.
who. Choice (A) associates insurance claim
with hospital bill. Choice (B) associates 21. (B) The dress is priced under $75. Choice
shipping with handles. (A) confuses the similar-sounding words
guess and dress. Choice (C) associates cost
12. (A) The clerk will assist a hotel guest
with price.
who loses a room key. Choice (B)
confuses the similar-sounding words use 22. (C) The second speaker has already seen
and lose. Choice (C) confuses the similar- John’s new car. Choice (A) associates
sounding words Turkey and key. garage with car. Choice (B) confuses
walking with the similar-sounding word
13. (B) Since the barber is free this morning,
talking.
he can cut the speaker’s hair then. Choice
(A) confuses the similar-sounding words 23. (A) The speaker is surprised to hear that
hair and air. Choice (C) confuses what the Mary will be taking yet another trip.
barber will cut. Choice (B) confuses the expression out of
town with downtown. Choice (C) confuses
14. (A) The French associate answers who.
weak with week.
Choice (B) confuses traveling to France
and translating the letter into French. 24. (B) The darker one is an import answers
Choice (C) associates travel agent with what is the difference between the cars.
French/France. Choice (A) associates power with cars.
Choice (C) confuses the similar-sounding
15. (C) The marketing department has a list
phrases between you and me and the
of suggestions for the name of the
difference between.
product. Choice (A) confuses call (to talk
on the telephone) and call (to name). 25. (C) I forgot to lock it up answers who and
Choice (B) does not answer the question. why. Choice (A) confuses lying (not
telling the truth) and lying (situated).
16. (B) The speaker responds to the
Choice (B) confuses everyone left and left
suggestion of eating with a reason for not
the money.
being hungry. Choice (A) confuses farther
with the similar-sounding word starving. 26. (B) Near the desks answers where. Choice
Copyright © 2012 by Pearson Education, Inc. Permission granted to reproduce for classroom use.
Choice (C) confuses seat with the similar- (A) does not answer the question. Choice
sounding word eat. (C) associates tables with chairs.
17. (B) I don’t have time answers why by 27. (C) It’s nine now answers what time is it.
providing a reason for not resting. Choices (A) and (B) repeat the word time
Choice (A) confuses the words rest (what but do not answer the question.
is left) and rest (sleep). Choice (C) 28. (A) I’d like some fish answers what would
confuses the similar-sounding words best you like for dinner. Choice (B) associates
and rest. eat with dinner and confuses the similar-
18. (C) The host will be the guest speaker. sounding words meet and eat. Choice (C)
Choice (A) confuses the similar-sounding associates cook with dinner.
words guess with guest and eat with 29. (B) To the local college answers where.
meeting. Choice (B) confuses the words Choice (A) does not answer the question.
bored and board. Choice (C) repeats the word school but
19. (A) The first speaker got home late at does not answer the question.
night, so the second speaker assumes the 30. (C) The conference room will be free
first speaker is very tired now. Choice (B) when the meeting is over; in an hour
repeats the phrase get in and confuses answers when. Choice (A) associates
flight with the similar-sounding word cost with free. Choice (B) associates spend
midnight. Choice (C) confuses the similar- with free.
sounding phrases get in and get one.
31. (C) They will meet in the second 40. (C) He decided to work at home answers
speaker’s office. Choice (A) confuses the why by providing a reason. Choice (A)
similar-sounding wh-questions when and repeats the word work but does not
where. Choice (B) associates directions answer the question. Choice (B)
with where. associates company and business
32. (C) The second speaker agrees that the with work.
weather is very nice. Choice (A) confuses
PART 3 (PAGES 227–229)
together with the similar-sounding word
weather. Choice (B) repeats the word 41. (A) The person who needs to hire a
pleasant. secretary is the one who is complaining.
Choice (B) can’t be correct because this
33. (B) The second speaker doesn’t yet have
person hasn’t even been hired yet.
a vacation start date. Choice (A) repeats
Choice (C) is associated with sick,
the word start but does not answer the
hospital, and nurse. Choice (D) is incorrect
question. Choice (C) confuses the
because that person is in the hospital.
similar-sounding words wait and date.
42. (B) The woman says that he is in the
34. (C) In the office answers where. Choice (A)
hospital. Choice (A) is mentioned in the
answers will you go. Choice (B) confuses
conversation, but it is not where the
the similar-sounding words reason and
secretary is. Choice (C) is where he will
season.
be tomorrow. Choice (D) confuses
35. (B) The report must be error-free answers nursing home with nurse and home.
why by providing a reason for the work
43. (C) The woman says that he has been in
to be checked. Choice (A) does not
the hospital for five days. Choice (A)
answer the question. Choice (C)
uses the word one in a different context.
associates the question with being a
Choice (B) confuses two days with the
problem.
number of secretaries normally in the
36. (C) In about a week answers when. Choice office. Choice (D) confuses week with
(A) confuses the similar-sounding words weak.
commuter and computer and associates on
44. (B) The woman is going through
time with when. Choice (B) associates
Copyright © 2012 by Pearson Education, Inc. Permission granted to reproduce for classroom use.
48. (D) The woman says that her brother 56. (C) This is when the woman says the
installed the dishwasher for her. Choice newspaper was delivered. Choice (A) is
(A) is a professional who would usually the opposite of the time it was delivered.
install dishwashers. Choice (B) is the Choice (B) confuses 4:00 with the similar-
person asking about the machine. Choice sounding word before. Choice (D)
(C) is who it was installed by. confuses 9:00 with the similar-sounding
49. (C) The woman says that it took 4 or 5 word mind.
hours. Choices (A), (B), and (D) sound 57. (B) The woman says that the office
similar to the correct answer. manager borrowed the newspaper.
50. (C) The survey was carried out at the Choice (A) is not mentioned. Choice (C)
time that the Olympics were on TV, associates delivery person with delivering
which was last month. Choice (A) is when the newspaper. Choice (D) confuses
the results of the survey were published. fiancé with the similar-sounding word
Choice (B) repeats last and week. Choice finance.
(D) repeats month and confuses three with 58. (C) Mr. Chen wants the financial section
33, which is the percentage of people of the newspaper back. Choice (A) is
watching the games. what the office manager did. Choice (B)
51. (A) This is what the woman says she did, repeats office manager. Choice (D) is what
and the man replies with facts that the office manager probably wants to do.
support her. Choices (B), (C), and (D) are 59. (C) The woman says we’ll close the shop for
contradicted by the survey results. two days after the holidays for inventory.
52. (D) This is the figure the man mentions. Choice (A) confuses Tuesday with the
The other choices sound similar to the similar-sounding phrase two days. Choice
correct answer. (B) repeats the word two. Choice (D)
associates the word vacation with
53. (C) The man says Just be careful to wear the holidays.
right shoes. Choice (A) is what the woman
doesn’t want to have to do. Choice (B) is 60. (A) The shop will be closed for two days
confused with the place where the man for inventory. Choices (B), (C), and (D)
goes jogging. Choice (D) is what the man sound similar to the phrase for two days.
Copyright © 2012 by Pearson Education, Inc. Permission granted to reproduce for classroom use.
does every morning, but he doesn’t 61. (D) The man mentions the office supplies
advise the woman to do this too. that will have to be counted during
54. (C) The man goes jogging in the field inventory. Choice (A) confuses clothes
behind the high school. Choice (A) confuses with the similar-sounding word close.
park with the similar-sounding word Choice (B) is not mentioned. Choice (C)
dark. Choice (B) uses the word course in a confuses books with the similar-sounding
different context. Choice (D) confuses word look.
pool with the similar-sounding word 62. (A) The woman is watching real estate
school. because she wants to get rich. Choice (B)
55. (A) The man says he will go jogging at confuses gold with the similar-sounding
5:30. Choice (B) and (C) are the times word old. Choice (C) is what the man
when the woman might go jogging. invested in. Choice (D) confuses art with
Choice (D) confuses eight with the the similar-sounding word smart.
similar-sounding word wait. 63. (C) The woman says I’ll watch it for a
month or two and then decide. Choice (A)
repeats the word risk. Choice (B)
associates banker with making
investments. Choice (D) repeats the word
market.
64. (C) The man says that he lost money in PART 4 (PAGES 230–232)
the stock market. Choice (A) associates
71. (D) The speaker says that the rainy
house with real estate. Choice (B) is what
season begins today. Choice (A) confuses
the woman says she wants to do. Choice
running with the similar-sounding word
(D) confuses art with the similar-
rainy. Choice (B) repeats the word home.
sounding word smart.
Choice (C) confuses train with the
65. (B) The woman says That’s the second time similar-sounding word rainy.
this year. Choice (A) confuses once with
72. (B) The afternoons will be dry and sunny,
the similar-sounding phrase once again.
which are pleasant conditions for
Choice (C) confuses three times with the
walking. Choices (A) and (C) are
similar-sounding word threatening.
contradicted by rains will begin again in
Choice (D) confuses five with fifty.
the evening and continue through the night
66. (B) The post office is threatening to raise and morning. Choice (D) is not
rates again in six months. Choice (A) mentioned.
confuses six days with six months. Choice
73. (B) People are advised to carry an
(C) associates two with second. Choice (D)
umbrella every morning. Choice (A) is
confuses another six years with another six
what the weather report is being aired
months.
on. Choices (C) and (D) are not
67. (A) According to the woman, it will still mentioned.
cost under 50 cents to mail a postcard.
74. (C) Reports were registered with the
Choice (B) repeats 50 cents. Choices (C)
police by two local residents. Choice (A)
and (D) are associated with the man’s
is not mentioned. Choice (B) associates
complaint that soon it will cost over a
flier with flying objects. Choice (D)
dollar to send a postcard.
associates two reporters with two reports.
68. (C) The new employee has been working
75. (B) The residents claimed to have seen
for the woman for a week. Choice (A)
three spaceships. Choice (A) associates
confuses two days with the similar-
fliers with space and flying. Choice (C)
sounding word today. Choice (B)
associates flew in space with spaceships.
confuses four with for. Choice (D) is the
Choice (D) is incorrect because the police
Copyright © 2012 by Pearson Education, Inc. Permission granted to reproduce for classroom use.
78. (B) The power company requests that 85. (D) The announcer says We will keep you
only businesses run their cooling systems posted on any further developments. Choice
in the afternoon. Choice (A) is (A) uses the word posted in a different
mentioned, but it is never said when they context by associating it with mail.
can run their cooling systems. Choice (C) Choice (B) repeats the phrase without
is requested to run cooling systems in the delay. Choice (C) uses the word develop
morning and evening. Choice (D) is in a different context.
associated with a restriction on power
86. (B) The mayor is making the
use but is contradicted by the correct
announcement. Choice (A) associates
answer.
high school with teenager and school.
79. (D) The power company is asking for full Choice (C) is where the concert will
cooperation. Choice (A) confuses the be held. Choice (D) is who will perform
similar-sounding words experience and the concert.
experiencing. Choice (B) confuses the
87. (B) The concert will take place at 8:00 P.M.
company’s asking for business and its advice Choice (A) confuses the time of the concert
to businesses. Choice (C) is the service the and working hard before school. Choice (C)
company provides. confuses 8:00 A.M. and 8:00 P.M. Choice
80. (C) A man, who was a dentist, got off the (D) confuses the time of the concert and
bus. Choice (A) associates doctor with school.
dentist. Choice (B) is who bit the man. 88. (D) There are only 500 seats in the
Choice (D) confuses the similar-sounding auditorium. Choice (A) is the number of
word accountant with the phrase by all hardworking youngsters who will be
accounts. performing the concert. Choices (B) and
81. (A) The man accidentally stepped on the (C) are the room numbers where you can
dog. Choice (B) is incorrect because the get tickets.
man bit the dog’s head, not leg. Choice 89. (A) The shaver has stainless steel blades.
(C) is not mentioned, but it is unlikely Choice (B) tells what is made of
the man would release his hold platinum. Choice (C) is not specified.
accidentally. Choice (D) is incorrect Choice (D) associates face with whiskers
because both the man and the dog
Copyright © 2012 by Pearson Education, Inc. Permission granted to reproduce for classroom use.
and shaving.
went to their respective doctors, but
the man probably did not take the dog 90. (C) The blades rotate. Choices (A), (B),
to the doctor. and (D) are contradicted by the cutters
rotate.
82. (A) The dog was angry because the man
stepped on it. Choice (B) is not 91. (C) The shaver costs $45.95. Choices (A),
mentioned. Choices (C) and (D) might be (B), and (D) all sound similar to the
true but are not the primary motivators correct answer.
behind the bite. 92. (A) The company is announcing a change
83. (A) The announcer was surprised at the in business hours. Choices (B) and (C)
report. Choices (B), (C), and (D) were not are possible things that a company might
mentioned, but could be common announce, but they are not mentioned.
reactions to a weather forecast. Choice (D) associates price with bought
and sold.
84. (B) This report is on the weather. Choices
(A), (C), and (D) are not mentioned. 93. (B) Investors in stock would be likely to
do business with a company that buys
and sells stocks and bonds. Choice (A)
associates jewelry with rock. Choice (C)
associates coin collector with currency
exchange. Choice (D) is not mentioned.
94. (C) The announcer says that the company PART 5 (PAGES 233–236)
will not be open on Saturdays or Sundays.
101. (B) As well as is a conjunction that adds
Choice (A) is the time when currency can
ideas. Choice (A) is a conjunction that is
be exchanged. Choice (B) is the day when
paired with neither. Choice (C) shows
the changes go into effect. Choice (D) is
contrast. Choice (D) requires a choice.
incorrect because the company is only
closed on certain days during these 102. (D) When is a conjunction that indicates
months. time. Choice (A) shows an unexpected
result. Choice (B) means at the same time.
95. (D) There will be a vacancy on the school
Choice (C) is an article.
board next May. Choices (A) and (C) are
where a job seeker can pick up an 103. Choice (A) is correct because it uses the
application. Choice (B) is who will past continuous in the passive. Choice
consider the application. (B) is in the passive, but it is the wrong
verb tense. Choices (C) and (D) are
96. (D) Anyone who is a resident of
incorrect because they are not in the
Smithtown and at least 21 years old may
passive.
apply. Choice (A) is contradicted by the
requirements mentioned. Choice (B) 104. (C) This real condition using the present
associates librarian with library. Choice tense requires the future tense in the
(C) confuses the similar-sounding words passive in the result clause. Choice (A) is
civil and civic. past tense in the passive. Choice (B) is
future tense, but it is active, not passive
97. (C) Interested people must fill out an
as needed. Choice (D) is present tense.
application for consideration by the city
council. Choice (A) is associated with 105. (A) The adverb of indefinite frequency
school board. Choices (B) and (D) are already indicates an action that is
associated with the places where completed. Choice (B) is incorrect
applications may be obtained. because adverbs of indefinite frequency
are not usually placed before the
98. (B) The captain of the plane is asking the
auxiliary verb. Choices (C) and (D)
passengers to get ready for landing.
have adverbs of indefinite frequency
Choice (A) associates theater with seats
that indicate pending rather than
Copyright © 2012 by Pearson Education, Inc. Permission granted to reproduce for classroom use.
109. (D) Thus is a transition word that 120. (B) Adverbs modify verbs. Choice (A) is
indicates cause and result. Choice (A) a noun. Choice (C) is an adjective. Choice
identifies an idea. Choice (B) is used to (D) is the past tense.
show an unexpected result. Choice (C)
121. (B) Consequently is a transition word that
adds an idea.
indicates a resulting idea. Choice (A)
110. (B) Yet is a transition word that indicates adds an idea. Choices (C) and (D) show
an unexpected result. Choices (A) and contrast.
(C) indicate cause and result. Choice (D)
122. (D) In advance means ahead of time.
indicates time.
Choices (A), (B), and (C) have meanings
111. (A) Personnel refers to employees and that don’t fit the context of the sentence.
describes the kind of department. Choice
123. (D) Accordingly is a transition word that
(B) is a plural noun. Choices (C) and (D)
indicates a result. Choice (A) indicates a
are adjectives.
contrast. Choice (B) is used with time.
112. (D) But is a conjunction that shows Choice (C) identifies an idea.
contrast. Choice (A) adds an idea. Choice
124. (B) Internal is an adjective that describes
(B) indicates cause and result. Choice (C)
the noun sources. Choice (A) is a present
indicates a result.
participle. Choice (C) is a noun referring
113. (C) Before is a conjunction that indicates to a person. Choice (D) is a past
in the time prior to. Choices (A) and (B) participle.
mean at the same time. Choice (D) means
125. (B) Causative made requires the simple
at a time later than.
form advance when the subject does the
114. (B) To is a preposition that means toward. action. Choice (A) is a past participle.
Choice (A) indicates origin. Choice (C) is Choice (C) is a present participle.
used with location. Choice (D) indicates Choice (D) is a noun.
an association.
126. (C) Look it over means to review. Choice
115. (A) With is a preposition that indicates an (A) means to search for information. Choice
association. Choice (B) indicates (B) means to begin to be interested. Choice
possession. Choice (C) means concerning. (D) means to remove.
Choice (D) means on behalf of.
Copyright © 2012 by Pearson Education, Inc. Permission granted to reproduce for classroom use.
130. (B) Use the present tense to describe a PART 6 (PAGES 237–240)
current situation. Choice (A) is the past
141. (A) As of followed by a date means
perfect. Choice (C) is the future tense.
beginning on the date mentioned. Choices
Choice (D) is the past tense.
(B), (C), and (D) are prepositions which
131. (C) Devote means give or commit. Choices do not follow at in this context.
(A), (B), and (D) have meanings that
142. (D) The paragraph outlines rules for
don’t fit the context of the sentence.
parking at the new location. Choice (B),
132. (D) In is a preposition used with cities. tickets, is mentioned later in the
Choice (A) indicates location or time. paragraph as a detail of the new rules.
Choice (B) indicates direction. Choice (C) Choices (A) and (C) are things associated
means on behalf of. with parking but are not the correct
133. (A) Adverbs of definite frequency can answer.
appear at the ends of sentences. Choices 143. (B) This is a negative imperative form,
(B), (C), and (D) are adverbs of indefinite telling customers that they are not
frequency. allowed to park in the all-day lot because
134. (B) Take on means to accept. Choice (A) it is for staff only. Choice (A) is the
means to comprehend. Choice (C) means present participle. Choice (C) is an
to be allowed to. Choice (D) means infinitive form. Choice (D) is the base
to wake up. form.
135. (C) Putting up means building. Choice (A) 144. (B) About is a preposition, and
means postponing. Choice (B) means prepositions are followed by gerunds.
dressing. Choice (D) means suggesting to. Choice (A) is the base form. Choice (C) is
a future verb. Choice (D) is an infinitive
136. (B) Although is a transition word that verb.
indicates a contrast. Choice (A) indicates
an unexpected effect, (C) indicates an 145. (A) Plenty of equipment means enough
expected effect, and (D) adds an idea. equipment. Choices (B), (C), and (D)
cannot be used with any meaning in this
137. (D) Or is a conjunction that excludes both context.
ideas. Choices (A) and (B) add ideas.
146. (D) A candidate is a person who may be
Copyright © 2012 by Pearson Education, Inc. Permission granted to reproduce for classroom use.
149. (A) This sentences refers to a specific 158. (D) Due to the increase in freight rates, it
web page, so the definite article the is is necessary to raise coffee prices. Choice
required. Choice (B) is incorrect because (A) is not mentioned. Choice (B)
the web page does not belong to the associates improved quality with high-
writer of the directions. Choice (C) is an quality. Choice (C) is not mentioned.
indefinite article. Choice (D) refers to all
159. (C) A cursory survey is one that is done
web pages, but these directions are about
quickly and superficially. Choices (A),
just one web page.
(B), and (D) do not fit the context.
150. (C) Whether introduces a choice. Choice
160. (C) The e-mail is probably written in
(A) introduces a time clause. Choice (B)
South America and being sent to
means in spite of. Choice (D) is an adverb
customers in Europe, the continent across
of frequency.
the Atlantic Ocean. Choices (A) and (B)
151. (B) Attached is an adjective describing the would be on the Pacific Ocean route.
noun terms. Choices (A) and (C) are Choice (D) would not be affected by
verbs. Choice (D) is a noun. increased Atlantic freight rates, since
152. (D) The recipient of the e-mail must Brazilian coffee is produced in South
America.
respond, or answer, in order to receive the
checks. Choices (A), (B), and (C) look 161. (B) Reservations are not required for
similar to the correct answer but don’t fit trains with unreserved coach service.
the context. Choices (A), (C), and (D) require
reservations.
PART 7 (PAGES 241–257)
162. (C) A service fee is charged when
153. (B) The 1995 ordinance imposed a height reservations are not used and not
limit. Choices (A), (C), and (D) are canceled. Choices (A), (B), and (D) are
contradicted by the information in not times when fees are charged.
the report.
163. (B) If the train arrives late, it will depart
154. (A) Through eminent domain, the city the station as soon as possible in order to
could now acquire neglected and stay on schedule. Choice (A) is
decrepit buildings. Choice (B) is not contradicted by advising passengers to
Copyright © 2012 by Pearson Education, Inc. Permission granted to reproduce for classroom use.
167. (B) The cost of the seminar is $35 per 176. (A) They can suffer from minor diseases
person. Choices (A), (C), and (D) are not like headaches to extremely bad (severe)
mentioned. cases of respiratory problems. Choices
168. (C) Ms. Ireland recommends that the (B), (C), and (D) do not fit the context.
company purchase the kit. Choice (A) is 177. (D) Employee absences due to illness
contradicted by the distinct mention of increased. Choice (A) refers to the time it
supervisory employees. Choice (B) is too takes to pass through double doors, but
specific; it refers to the recipient of the this is not mentioned as a problem.
e-mail message. Choice (D) is a result of Choice (B) refers to the energy-saving
attending the seminar. measures. Choice (C) refers to the
contaminants that remain in the air.
169. (B) The other employees will probably be
trained in an in-house workshop held by 178. (A) Employees began to notice an
Ms. Ireland. Choices (A), (C), and (D) are increase in headaches. Choices (B), (C),
contradicted by Ms. Ireland’s and (D) are not mentioned.
recommendation. 179. (B) The people who work for Tom are his
170. (A) Congenial means friendly, able to get staff members. Choice (A) uses the word
along well with people. Choices (B), (C), members in a different context. Choice (C)
and (D) are not mentioned. is incorrect because Gloria does not
mention her friends in the message.
171. (D) Mr. Proctor has experience as a clerk
Choice (D) is incorrect because the
in the sporting goods section of a
stadium is mentioned but stadium
department store. Choice (A) associates
employees are not.
gourmet with plate, by confusing the
similar-sounding word La Plata. Choices 180. (C) Gloria wrote Anyone who is interested
(B) and (C) are not mentioned. in these tickets should contact me by e-mail.
Choices (A) and (B) are not mentioned.
172. (A) The position Mr. Proctor is applying
Choice (D) uses the word fair in a
for is in Santa Fe, New Mexico. Choice
different context.
(B) is where Mr. Proctor worked with Mr.
Wann. Choice (C) is where Mr. Proctor 181. (B) On the invoice, Ari Gupta charged for
used to live. Choice (D) is the location of writing a monthly column and a special
Copyright © 2012 by Pearson Education, Inc. Permission granted to reproduce for classroom use.
the store where Mr. Proctor worked with feature article. Choices (A) and (D) are
Mr. Wann. associated with money and invoices.
Choice (C) associates news reporting
173. (B) The report is needed January 6.
with writing.
Choice (A) is the date the message was
taken. Choice (C) confuses the date with 182. (D) In his letter. Mike Wallace apologizes
the time the message was taken. Choice for paying a week late. Choice (A) is
(D) is contradicted by tomorrow. contradicted by the correct answer.
Choice (B) is the date of the invoice.
174. (A) The message is from James Bryant.
Choice (C) is not mentioned.
Choice (B) is who the message is for.
Choice (C) is who took the message. 183. (A) Mike Wallace was charged $985 to
Choice (D) is contradicted by the name which he added a 2% late fee of $19.70,
on the “from” line. bringing his total payment to $1004.70.
175. (C) Construction of office buildings Choice (B) was the amount he was
changed to deal with the high energy charged for editing services only. Choice
costs related to heating and cooling (C) is the total amount he was charged
offices. Choices (A), (B), and (D) are not for the special feature. Choice (D) is the
mentioned. total amount he paid.
184. (A) Mike Wallace’s letter says that he 190. (D) In the e-mail message, Alexia says
likes the new payment plan and that it is she wants to try offering cheaper tickets
easier. Choice (B) is incorrect because he on Thursday. Choice (C) is the day she
didn’t ask any questions about it. Choice says she doesn’t want to do this. Choices
(C) is incorrect because he paid the late (A) and (B) are not mentioned.
payment fee without making any
191. (A) In his letter, Kim says Unfortunately,
complaint. Choice (D) is incorrect
we are not in the position to offer you a raise
because he says he actually prefers the
at this time, so we can assume that Choi
new plan to another form of payment
has asked him for a raise. Choice (B) is
(credit card).
not mentioned; in fact Kim seems to
185. (B) The client’s check has to clear want Choi to stay at his job. Choice (C) is
through the bank before Gupta can use incorrect because, even though Kim
the money. Choice (A) is contradicted by mentions the possibility of a promotion
the correct answer. Choice (C) is in the future, he does not offer one now.
incorrect because it is Wallace, not Choice (D) is incorrect because, even
Gupta, who is not a verified Payday Now though Kim seems to think that Choi is
user. Choice (D) confuses the check with doing a good job, he never thanks or
the invoice. praises him for it.
186. (C) The article explains that watching 192. (D) We understand from Kim’s letter that
movies at home is cheaper than taking a Choi had complained of his heavy
family to the movie theater. Choice (A) is workload, and Kim attributes this to
mentioned as a reason to go to a movie Choi’s having taken on the
theater, not to avoid one. Choice (B) is responsibilities of a former employee.
incorrect because the quality of DVDs Choices (A), (B), and (C) are possible
watched at home is not mentioned. results of a heavy workload but are not
Choice (D) is not mentioned. mentioned.
187. (A) Perry Scott is the owner of the 193. (C) Kim says that no one will get a salary
London Cinema. Choice (B) refers to Earl raise because of low sales. Choice (A) is
Hutton, the recipient of the e-mail. not true because Kim has plans for
Choice (C) is associated with snacks. increasing the company’s sales and store
Copyright © 2012 by Pearson Education, Inc. Permission granted to reproduce for classroom use.
Choice (D) is associated with movies and hours. Choice (B) is not mentioned.
movie theaters. Choice (D) is not true because Choi has
188. (B) Many movie theaters have increased taken on a heavy workload.
their snack prices by 50%, and Alexia 194. (A) In his letter, Choi says that he is
wants to increase prices by 20% less than resigning from his position as sales
that. Choice (A) is the decrease in profits supervisor. Choice (B) is Kim Moon’s
mentioned by Perry Scott. Choice (C) is position. Choice (C) is the future position
the average increase in snack prices. mentioned in Kim’s e-mail. Choice (D) is
Choice (D) is 20% more, not less, than the Choi’s new position.
average increase in snack prices.
195. (A) Choi says he may reapply when the
189. (A) Several things are mentioned in the new position becomes available; he is
article, and Alexia likes the first one, referring to the position of marketing
which is drinks. Choices (B), (C), and (D) director mentioned in Kim’s e-mail.
are the other items mentioned in the Choice (B) is confused with the person
article. who quit last year. Choice (C) is
something Choi has already done.
Choice (D) is not mentioned.
196. (D) This is the date mentioned on the 199. (D) The customer is directed to pick up
search form. Choice (A) is not his car from HK Mobiles, which is
mentioned. Choice (B) is when he located 5 kilometers from the airport.
planned to pick up the car. Choice (C) is Choice (A) mentions the Arrivals level,
when he actually will drop off the car. which is where the customer can get a
197. (C) For Rental agency, the customer chose shuttle to the agency. Choice (B) is
Find me the best deal, so he didn’t ask for a mentioned, but it is not called HK
specific agency but for the cheapest one. Mobiles. Choice (C) is incorrect because
Choices (A), (B), and (D) are things that the type of vehicle the customer selected
the customer specified. is a mini-van.
198. (A) The rental confirmation says that a 200. (B) Of all the things the customer
customer will receive a 20% discount if originally asked for, this is the only item
he returns the car within six days, so the that appears on his booking form.
customer changed his original plan of Choices (A), (C), and (D) are things that
keeping the car seven days. Choice (B) is the customer originally selected but that
mentioned, but there is no time don’t appear on the booking form.
restriction for it. Choice (C) is incorrect
because the customer still has to pay a
deposit. Choice (D) is incorrect because
the customer is actually getting a smaller
car than he originally asked for.
Copyright © 2012 by Pearson Education, Inc. Permission granted to reproduce for classroom use.
ANSWER KEY
PRACTICE TEST TWO
PART 1 (PAGES 259–264) 7. (C) Groups of people are admiring
paintings in a gallery of an art museum.
1. (A) The man is reading a newspaper.
Choice (A) misidentifies the location.
Choice (B) associates journalist and story
Choice (B) mentions the walls and uses
with newspaper and confuses a journalist
the associated word display. Choice (D)
covering a story (writing an article) and
uses the associated words artist and
newspaper covering the table. Choices (C)
portrait.
and (D) misidentify the person and
the action. 8. (C) Three business colleagues are seated
at a conference table and looking at a
2. (B) The woman is injecting liquid into a
map displayed on a screen. Choice (A) is
glass tube. Choice (A) uses a word
incorrect because no one in the picture is
associated with liquid: spill. Choice (C)
pointing at the map. Choices (B) and (D)
associates water with liquid. Choice (D)
mention the table but no one is polishing
uses the associated word filling.
it or setting it.
3. (B) The mechanics are repairing the
9. (A) Women are sewing clothes in a
airplane engine. Choice (A) is incorrect
factory. Choice (B) mentions the clothes,
because the plane cannot take off until it
but no one is trying them on. Choice (C)
is fixed. Choice (C) confuses car motor
confuses similar-sounding words clothes
and plane engine. Choice (D) confuses a
and close. Choice (D) mentions the thread
bird’s wing and the plane’s wing.
but no one is buying it.
4. (C) The newspapers are displayed in a
10. (C) The room is full of tables set for a
rack. Choice (A) confuses the similar-
meal, but there is no one at the tables.
sounding words paper and newspaper and
Presumably the event has not started yet.
Copyright © 2012 by Pearson Education, Inc. Permission granted to reproduce for classroom use.
14. (B) The second speaker wants to leave 23. (A) In two hours answers when the movie
earlier to avoid the heavy afternoon will be shown a second time. Choice (B)
traffic. Choice (A) associates weight with incorrectly answers where. Choice (C)
heavy. Choice (C) confuses spoon with the confuses the similar-sounding words fee
similar-sounding word afternoon. and movie and the words for and four.
15. (C) The committee meets in the 24. (B) A hamburger is a type of sandwich.
conference room. Choice (A) uses the Choice (A) confuses the words type
preposition in, which is also used with (typewriting) and type (kind). Choice (C)
location but answers when the committee answers with a preference but does not
meets. Choice (B) answers how often the answer the question.
committee meets.
25. (C) The second speaker expresses an
16. (B) Plumbers install and repair water opinion after hearing the price of the
fixtures and fix leaks. Choice (A) shoes. Choice (A) confuses news with the
confuses the similar-sounding words heat similar-sounding word shoes. Choice (B)
and leak. Choice (C) confuses the similar- associates bank account with the mention
sounding words week and leak. of money.
17. (B) Because I don’t like the sun provides a 26. (B) Sorry is a polite way to refuse a
reason for drawing the shades. Choice suggestion. Choice (A) confuses the
(A) incorrectly associates too long with similar-sounding phrases look out and
drawn. Choice (C) does not answer why. going out. Choice (C) confuses the
18. (A) The speaker wants to know who sent similar-sounding words lungs and lunch.
the package. Choice (B) answers a 27. (C) No, I’ve never been there is a logical
different question. Choice (C) confuses response to a yes/no question. Choice (A)
packing with the similar-sounding word confuses the similar-sounding words tie
package. with Thailand. Choice (B) associates room
19. (C) This is a proper response to the with view and visited.
announcement of an out-of-town trip. 28. (A) No, most of them are Latin American is
Choice (A) confuses downtown with the a logical response to a yes/no question.
similar-sounding phrase out of town. Choice (B) confuses the similar-sounding
Copyright © 2012 by Pearson Education, Inc. Permission granted to reproduce for classroom use.
Choice (B) repeats the word tomorrow. words aching and Asian. Choice (C)
20. (B) We’ll probably eat around six answers associates list and buyers with clients.
when. Choice (A) confuses the similar- 29. (C) I’ll see if we have them is the best
sounding phrases going with you and response to the request. Choice (A)
going to eat. Choice (C) provides the time confuses the similar-sounding words eyes
of a meeting, not the time for dinner. and size and associates eyes with pair.
21. (B) Three hundred and fifty dollars is how Choice (B) associates feet and shoes.
much the printer costs. Choice (A) 30. (A) It’s hers answers whose. Choice (B)
incorrectly answers where. Choice (C) answers which. Choice (C) does not
associates inkjet model with printer. answer the question.
22. (A) Because usually states a reason that 31. (B) We don’t have a listing is the best
answers a why question. Choice (B) response to the question. Choice (A)
confuses the words change (to substitute) associates books and publishing. Choice
and change (money). Choice (C) confuses (C) does not answer the question.
the similar-sounding words broken
and broker.
32. (C) It’s two blocks that way answers how PART 3 (PAGES 266–268)
far. Choice (A) associates mail and post
41. (B) The man says the accountant called.
office but confuses the similar-sounding
Choice (A) confuses assistant with the
words a day and away. Choice (B)
similar-sounding word accountant.
confuses the similar-sounding words
Choice (C) is the person whose forms the
posted and post office.
accountant wants to see. Choice (D)
33. (B) It usually is is the best response to Will confuses the word manager with manage.
the bus be on time. Choice (A) gives the
42. (A) The man asks the woman to check
time. Choice (C) associates watch
some figures, and she agrees to do so.
and time.
Choice (B) confuses delivering mail with
34. (C) I did answers who. Choice (A) the woman’s mailing the forms after she
confuses the words cold (illness) and heat. checks the figures. Choice (C) associates
Choice (B) does not answer the question. bank with figures. Choice (D) uses the
35. (B) Forty-five people answers how many. word check in a different context.
Choice (A) confuses the similar-sounding 43. (A) The woman says she will check the
words easy and employees. Choice (C) figures right now. Choice (B) confuses
associates labor union with employees. tonight with the similar-sounding word
36. (A) Jazz is a type of music. Choice (B) right. Choice (C) is when the figures will
confuses the words kind (nice) and kind be mailed. Choice (D) is when the
(type). Choice (C) confuses the similar- accountant will receive them.
sounding words sick and music. 44. (B) The man is asking for information
37. (C) The cleaning service didn’t come about a museum and a bus schedule, so
answers the question why. Choice (A) is he must be a tourist. Choice (A) confuses
incorrect because isn’t clean means the folk dancer with folk museum. Choice (C)
same as dirty, but the answer refers to a associates hotel worker with hotel. Choice
shirt, not a place. Choice (B) associates (D) associates bus driver with bus.
tidy and clean; picking up toys would 45. (A) The bus that will take him to the
make the place tidy but not clean. museum stops in front of a hotel. Choice
38. (A) Your son’s teacher called is the (B) is incorrect because the bus stops one
Copyright © 2012 by Pearson Education, Inc. Permission granted to reproduce for classroom use.
message. Choice (B) confuses the similar- block away from the museum. Choice (C)
sounding words missed and messages. is where the man is now. Choice (D) uses
Choice (C) confuses the similar-sounding the word park in a different context.
words message and package. 46. (D) The woman tells the man that the
39. (B) The bullet train should be the fastest bus runs every 30 minutes. Choices (A)
(quickest). Choice (A) associates express and (C) confuse the time the next bus
with quickest and incorrectly gives may leave. Choice (B) sounds similar to
information on the train schedule. the correct answer.
Choice (C) associates tracks with train. 47. (B) The woman says that she is an
40. (A) The speaker explains why George agricultural specialist. Choice (A)
hasn’t been in the office lately. Choice (B) associates hotel with stay. Choice (C) is
repeats the word office and confuses later not mentioned. Choice (D) associates
with the similar-sounding word lately. ambassador with embassy.
Choice (C) repeats the word office. 48. (A) The woman says that she is working
temporarily at the embassy. Choices (B)
and (D) are things the man suggests that
she do in addition to working. Choice (C)
confuses meeting with the similar-
sounding word sightseeing.
49. (C) The woman says that she will stay for 57. (B) The woman submitted the report on
six months altogether. Choice (A) is how Monday. Choice (A) confuses Sunday
long she has already been here. Choice with the similar-sounding word Monday.
(B) is how much longer she will stay. Choice (C) confuses week with weak.
Choice (D) confuses ten with the similar- Choice (D) confuses month with the
sounding word then. similar-sounding word Monday.
50. (C) The woman says I must’ve left it at 58. (A) The man says that the report is weak,
home. Choice (A) is how she will get doesn’t have much information, and is
downtown. Choice (B) is where she disorganized. Choices (B) and (C) are the
looked for the wallet. Choice (D) opposite of the man’s opinion. Choice
associates bank with lend money. (D) confuses Mr. Frank with frankly.
51. (D) The man offers to lend the woman 59. (D) The man decides to drive to Rome,
some money for a taxi. Choice (A) which will take a full day. Choice (A) is
repeats the word downtown. Choice (B) the length of the trip by plane. Choice (B)
repeats the word wallet. Choice (C) confuses five with the similar-sounding
confuses taxes with the similar-sounding word drive. Choice (C) confuses nine with
word taxi. the similar-sounding word fine.
52. (B) The woman says she has to be at a 60. (B) The man says that he wants to enjoy
meeting downtown in 15 minutes. the scenery. Choice (A) is incorrect
Choice (A) is where she left her wallet. because flying is faster than driving.
Choice (C) uses the word course in a Choice (C) repeats the word flying.
different context. Choice (D) confuses Choice (D) repeats the word reserve.
out of town with downtown.
61. (C) It costs $150 a day to rent a car.
53. (A) The speakers mention a delayed Choices (A), (B), and (D) sound similar
flight and the duty-free shop, so they to the correct answer.
must be at an airport. Choice (B)
62. (C) Ms. Carrera went to the airport to
associates restaurant with get a bite to eat.
pick up a package. Choice (A) is incorrect
Choice (C) associates shopping district
because the woman says that Ms. Carrera
with buy and shop. Choice (D) associates
is out of the office. Choice (B) confuses
Copyright © 2012 by Pearson Education, Inc. Permission granted to reproduce for classroom use.
66. (A) The man is looking for the tea. 74. (A) The purpose of the announcement is
Choice (B) repeats the word coffee. Choice to thank the hosts for their hospitality.
(C) confuses book with the similar- Choice (B) is not mentioned. Choice (C)
sounding word look. Choice (D) uses the associates donations with offering their
word show in a different context. home. Choice (D) is not mentioned.
67. (B) The woman will show the man where 75. (B) The Spring Gala is in three weeks.
the tea is. Choice (A) confuses shower Choice (A) confuses lunch and week with
with the similar-sounding word show. the date of the gala. Choice (C) associates
Choice (C) confuses ready with the next spring with Spring Gala. Choice (D)
similar-sounding word already. Choice associates spring break with Spring Gala.
(D) repeats the word coffee. 76. (D) The Spring Gala will be held at the
68. (B) The woman says I’m getting the gray Forest Hills Resort. Choice (A) associates
one. Choice (A) is the coat she won’t buy. embassy with consul general. Choice (B) is
Choice (C) confuses white with the incorrect because today’s luncheon was
similar-sounding word right. Choice (D) held at the consul general’s home, but
confuses blue with the similar-sounding the gala will not be held there. Choice (C)
word new. associates campground with outdoor events
and resort.
69. (A) The man says that the gray coat fits
the woman better, and she agrees with 77. (B) The Cineplex is showing a double
him. Choices (B) and (C) describe the feature, which means two films. Choice
black coat. Choice (D) uses the word (A) confuses one film with one feature.
better, but in a different context. Choices (C) and (D) are the times when
the double feature is being shown.
70. (D) The coat costs $860. Choices (A), (B),
and (C) sound similar to the correct 78. (D) Speeding Faster is about the lives of
answer. two female race car drivers. Choice (A) is
not mentioned. Choice (B) confuses the
PART 4 (PAGES 269–271) two movies; Moonlight over Lisbon is
71. (C) According to the advertisement, there about a classical guitarist. Choice (C)
are special prices on all shoes and clothing in confuses the similar-sounding words
Copyright © 2012 by Pearson Education, Inc. Permission granted to reproduce for classroom use.
the store. Choice (A) repeats the word starving (hunger strike) and starring.
home. Choice (B) is incorrect because 79. (A) Matinees are offered on Saturdays
although there is a special reduction on and Sundays, which means the same as
coats, they are not the only items on sale. the weekend. Choices (B), (C), and (D)
Choice (D) is contradicted by the correct are not mentioned as days matinees
answer. are shown.
72. (D) Women’s coats cost $250. Choices (A) 80. (C) The announcement is about an art
and (B) sound similar to the correct auction. Choice (A) confuses labor hall
answer. Choice (C) is confused with the with labor law. Choice (B) is incorrect
$25 dollar down payment. because it names the sponsors, not the
73. (B) The advertisement says that the purpose. Choice (D) associates weather
special offer is for customers with a with White Cloud.
Maury’s credit card. Choice (A) repeats
the words new and customers. Choice (C)
repeats the word women. Choice (D)
repeats the word today.
81. (A) Art buyers would be most interested 87. (A) Civil war destroyed much of
in attending an art auction. Choice (B) is Colberia’s economy. Choice (B) is
incorrect because although artists may be incorrect because businesspeople left the
interested in the auction, art buyers are country as a result of the failed economy
the primary audience. Choice (C) and civil war. Choice (C) is not
associates petrochemical workers with mentioned. Choice (D) confuses natural
firefighters. Choice (D) incorrectly disaster with Colberia’s natural resources.
assumes victims of home fires would be (C) The country had been a producer and
88.
the most interested in buying art to
exporter of natural resources (timber, oil,
benefit fire safety.
and agricultural products). Choices (A),
82. (B) The annual art auction will be held at (B), and (D) are not mentioned.
the Labor Hall of Ontario. Choice (A)
89. (D) The message is being played because
incorrectly assumes that because dinner
it is after office hours. Choice (A) is
will be served that the event will be held
incorrect because the message does not
at a restaurant. Choice (C) incorrectly
mention the number being out of service.
assumes that, because the Firefighters
Choice (B) is not mentioned. Choice (C)
Association is holding the event, it will
confuses the computer being down and
be held at a firehouse. Choice (D)
the name of the company, Mainframe.
incorrectly assumes that, because the
event is an art auction, it will be held at a 90. (C) The caller is to leave a message, and
studio. the call will be returned on the next
working day. Choices (A), (B), and (D)
83. (B) The high temperature will be 34
are not mentioned.
degrees. Choices (A), (C), and (D) sound
similar to the correct answer. 91. (A) The office closes at 12:00 noon on
Saturdays. Choice (B) is incorrect
84. (A) The rain is described as freezing.
because the office is open from 9:00 to
Choice (B) is not mentioned. Choice (C)
6:00 on Monday. Choice (C) is incorrect
confuses the similar-sounding words
because the office opens at 9:00 A.M. on
slick and sleet. Choice (D) is incorrect Saturday. Choice (D) is incorrect because
because dreary describes the day, not the the office is open on Friday afternoon
rain.
Copyright © 2012 by Pearson Education, Inc. Permission granted to reproduce for classroom use.
95. (D) According to the report, 98 percent of 102. (A) Expertise should be modified by the
the children in the study have used a adjective technical. Choice (B) is a noun.
computer at least one time. Choice (A) Choice (C) is a noun referring to a
repeats the word repair. Choice (B) person. Choice (D) is the plural form
associates homework with home and school of a noun.
children. Choice (C) confuses owning a 103. (B) The causative verb tell takes the
computer for two years with a two-year infinitive not to smoke. Choice (A) is the
study. present tense. Choice (C) is the past
96. (C) According to the report, 85 percent of participle. Choice (D) is the present
wealthier children own a computer. continuous.
Choice (A) confuses percent with the 104. (A) The coordinate conjunction but
ages of the children in the study (6 to 18). requires an idea that contrasts with
Choice (B) is the percentage of low- wasn’t home: was able to contact him.
income children who own a computer. Choices (B) and (C) require the past
Choice (D) is the percentage of children participle contacted. Choice (D) does not
in the study who have used a computer. contrast with wasn’t home when I called.
97. (A) The report mentions some possible 105. (C) Looking forward to means anticipating
reasons why low-income children use with pleasure. Choice (A) means admiring
their computers less often and then says a person’s qualities. Choice (B) means
The foundation plans to look into these trying to find something. Choice (D) means
reasons in the next study. Choice (B) trying to locate information in reference
repeats the phrase Internet connections. materials.
Choice (C) is part of the subject of the
current study. Choice (D) repeats the 106. (D) Has been drinking suggests an action
word school. that starts in the past and continues in
the present. Choice (A) suggests an
98. (A) A professor is talking to the class action completed in the future. Choice
about an assignment. Choices (B), (C), (B) suggests an action completed in the
and (D) are associated with the situation past. Choice (C) suggests an action that
but are not the correct answer. will start in the future.
Copyright © 2012 by Pearson Education, Inc. Permission granted to reproduce for classroom use.
99. (D) The speaker says that the assignment 107. (A) Still can precede a negative form.
is due on Wednesday. Choice (A) Choices (B), (C), and (D) are adverbs that
confuses Sunday with the similar- cannot precede negative forms.
sounding word Monday. Choice (B) is the
day that the speaker is speaking. Choice 108. (C) Uncomfortable means lacking in
(C) confuses Tuesday with the similar- comfort. Mr. Chou doesn’t feel good
sounding phrase two days. sitting in economy class seats during
long flights. Choices (A), (B), and (D)
100. (D) In inviting the students to ask have meanings that don’t fit the context
questions, the speaker says the easiest way of the sentence.
to reach me is to talk to me after class.
Choice (A) repeats the word library. 109. (A) The causative verb had requires the
Choices (B) and (C) are other ways of simple form of the verb pick up. Choice
contacting the professor. (B) is the present tense. Choice (C) is the
past tense. Choice (D) is the gerund.
PART 5 (PAGES 272–275) 110. (C) During is a preposition meaning at the
101. (C) If people have already met, you do same time. Choice (A) is not possible
not have to introduce them. Choices (A), because while must be followed by a
(B), and (D) incorrectly suggest that an subject and a verb. Choices (B) and (D)
introduction is required. are prepositions but are not logical in the
sentence.
111. (C) Or is a conjunction that indicates a 120. (A) The applicant caused the vice
choice among items. Choice (A) indicates president to be delighted, so the past
a contrast between items. Choices (B) participle is used. Choice (B) is the
and (D) join clauses, not words. present participle. Choice (C) is an
112. (B) Past tense won in the if-clause adjective. Choice (D) is the simple form
requires would in the remaining clause. of the verb.
Choices (A) and (C) do not use would. 121. (C) The two-word verb worry about is
Choice (D) uses would but in the past followed by the gerund. Choice (A) uses
continuous tense. the wrong preposition. Choice (B) is the
113. (A) Even though suggests a contrast infinitive. Choice (D) is a preposition
between items. Choice (B) suggests a and object.
cause-and-effect relationship. Choice (C) 122. (B) Revise means to update or change.
indicates a contrast but should begin the Choice (A) means to give advice. Choice
main clause. Choice (D) suggests a cause- (C) means to develop a solution for a
and-effect relationship. problem. Choice (D) means a tool or small
114. (C) Replace means substitute or find machine.
something to take the place of. Before the 123. (D) The conjunction as soon as begins and
end of the year, the company will buy ends with as. Choice (A) is an adverb.
new computers to use in place of the old Choices (B) and (C) are incomplete forms
ones. Choices (A), (B), and (D) look of as soon as.
similar to the correct answer but have
124. (D) And is a coordinate conjunction that
meanings which don’t fit the context of
joins two ideas. Choice (A) is usually
the sentence.
used with or. Choice (B) suggests a choice
115. (A) The causative verb let requires the between items. Choice (C) suggests a
simple form of the verb eat. Choice (B) is cause-and-effect relationship.
the infinitive. Choice (C) is the gerund.
125. (D) Adverbs of definite frequency may
Choice (D) is the past tense.
occur at the end of a clause. Choices (A)
116. (D) Expand means to extend the company’s and (C) are adverbs of indefinite
business. Choice (A) can refer to prices or frequency. Choice (B) is an adverb of
Copyright © 2012 by Pearson Education, Inc. Permission granted to reproduce for classroom use.
costs, but not to operations. Choice (B) indefinite frequency but is used with a
implies an unnecessary or unwanted negative verb.
increase. Choice (C) means to make
126. (B) An occupant is someone who occupies
something better, not necessarily larger.
a place, or a tenant. The people who are
117. (A) The causative verb watch requires the in the office now will move out, and then
simple form of the verb repair. Choice (B) we can move in. Choices (A), (C), and (D)
is the past tense. Choice (C) is the have meanings that don’t fit the context
present tense. Choice (D) is the past of the sentence.
perfect tense.
127. (C) Because of is followed by a noun
118. (C) The past tense got suggests a phrase. Choices (A), (B), and (D) must be
completed past action. Choice (A) is the followed by clauses.
present tense. Choice (B) is the past
128. (D) The verb try can be followed by a
participle. Choice (D) is the past perfect.
gerund. Choice (A) is the simple form.
119. (C) Will increase indicates a possible Choice (B) is the past tense. Choice (C) is
future action. Choice (A) suggests an the present tense.
action that will be completed in the
129. (D) An action in the immediate future
future. Choices (B) and (D) must be used
can use the present tense. Choice (A) is
with a past tense verb in the if-clause.
the future tense. Choice (B) is the future
perfect tense. Choice (C) is the present
continuous tense.
130. (C) Despite is a preposition and can be 140. (A) Overheard means heard by accident.
followed by a noun phrase. Choices (A), Choice (B) means conquered. Choice (C) is
(B), and (D) are conjunctions and must be an adjective that means very upset.
followed by a clause. Choice (D) means took too much money
131. (C) Comparisons between two things use from a bank account.
an -er or more form of the adjective plus
PART 6 (PAGES 276–280)
than. Choice (A) is used on both sides of
the adjective in equal comparisons. 141. (B) A noun is required in this position as
Choice (B) is not used to indicate the object of the verb had. Choices (A)
comparison. Choice (D) may be used and (D) are adjectives. Choice (C) is a
with the adjective different but not with noun, but it refers to a person, not an
other comparisons. event.
132. (A) Access means entry. A password is 142. (C) The present participle of the verb
needed to enter or view the website. completes the present perfect continuous
Choices (B), (C), and (D) have meanings form, describing an action that has
that don’t fit the context of the sentence. occurred over the past year. Choice (A) is
a base form. Choice (B) is the past
133. (B) Not only . . . but also is a paired
participle. Choice (D) is the infinitive
conjunction. Choices (A), (C), and (D) are
form.
not paired with not only.
143. (B) The performers in the film don’t
134. (A) Ran into means encountered or
memorize scripts and are filmed in their
discovered. Choice (B) means accumulated
natural surroundings; that is, they don’t
debts. Choice (C) means used a supply of
practice or rehearse before filming.
something. Choice (D) means tried to get
Choices (A), (C), and (D) are words
elected to political office.
associated with filmmaking but don’t fit
135. (B) Adverbs of definite frequency may the context.
appear at the end of a clause. Choices
144. (B) Naoko wants everyone to come to the
(A), (C), and (D) place the adverb in
office to get their new cell phones before
incorrect positions.
the service on their old phones is
136. (C) Past continuous action can be
Copyright © 2012 by Pearson Education, Inc. Permission granted to reproduce for classroom use.
148. (A) The base form of the verb follows to 157. (C) Mr. Kim asks the staff to fill in the
to make the infinitive form. Choice (B) is attached schedule. Choice (A) is incorrect
past tense. Choice (C) is a present because Mr. Kim asks the staff to fill out
participle. Choice (D) is a noun. the schedule in order to prepare for the
meetings; he does not ask them to
149. (B) The guards will have to reactivate the
prepare a meeting. Choice (B) is incorrect
power, or turn it on again, if the power
because Mr. Kim is going to a conference,
fails, or stops. Choices (A), (C), and (D)
but he does not ask others to go. Choice
are words associated with electricity, but
(D) is incorrect because although the
they cannot be correctly used in this
meeting will deal with work-related
context.
stress, he does not ask the employees to
150. (B) The article is about getting payment be aware of it.
for bills that are late, or overdue. Choices
158. (B) A time that is committed is one that
(A), (C), and (D) could refer to bills but
has been previously scheduled. Choices
don’t fit the topic of the article.
(A), (C), and (D) do not fit the context.
151. (B) A gerund is required to act as the
159. (D) The fall is their least busy time of
subject of the sentence. Choice (A) is a
year. Choice (A) is incorrect because the
base form verb. Choice (C) is an
meetings wouldn’t be held if Mr. Kim
infinitive form. Choice (D) contains a
couldn’t attend. Choices (B) and (C) are
modal.
not mentioned.
152. (A) The reference is to the article at hand.
160. (B) DEXREADY provides instant access
Choice (B) would refer to a different
to Pacific Rim economic news. Choices
article. Choice (C) is a possessive form.
(A), (C), and (D) are ways to access the
Choice (D) is an object pronoun.
information.
PART 7 (PAGES 281–299) 161. (D) Since much of the information is
153. (D) This passage gives instruction for international and governmental, a
using a computer typing tutorial. Choice government policy advisor would
(A) confuses the related words television probably benefit most from the index.
and screen. Choice (B) associates à la carte Choice (A) associates computer
Copyright © 2012 by Pearson Education, Inc. Permission granted to reproduce for classroom use.
menu with a computer menu item. Choice programmer with computer program.
(C) associates a row of homes with Choice (B) associates television reporters
home row. with the news publications offered. Choice
(C) associates publishing executive with
154. (B) The exercise explains the home row publications.
and finger locations, which are the basics
of typing. Choices (A), (C), and (D) are 162. (A) The information would most likely
not mentioned. appear in a daily newspaper. Choice (B)
confuses a tax form with the topic of the
155. (A) Novice computer typists would find article. Choice (C) is incorrect because
this typing tutorial information useful. information on taxes would not be found
Choice (B) associates computer in a science journal. Choice (D) confuses
programmer with computer program. a law enforcement magazine and failure to
Choice (C) is not a logical choice. Choice enforce tax laws.
(D) associates school administrators with
typing instructor. 163. (B) The main idea of the passage is that
Havaria’s tax system needs to be
156. (A) The communication is addressed to changed. Choice (A) is not mentioned.
administration. Choices (B), (C), and (D) Choice (C) is an assumption but not the
associate human resources, planning, and main idea. Choice (D) is the desired
development with Mr. Kim’s attending the result of the changes suggested.
conference on human resources
development.
164. (A) A complicated tax structure is a 174. (D) Something found in abundance is
complex one, one that is difficult to something found in great quantity.
understand. Choices (B), (C), and (D) do Choices (A), (B), and (C) do not fit the
not fit the context. context.
165. (A) The article investigated the 175. (A) There are often seasonal variations in
informational flow from the world’s hotel prices at resorts. Choices (B), (C),
stock markets to the Baltic markets. and (D) are all possible causes for price
Choices (B), (C), and (D) are all increases but are not mentioned.
mentioned but are not the primary focus.
176. (B) Travelers are often offered discounts
166. (A) Baltic stock markets follow when they make their reservations
international markets. Choices (B), (C), online. Choice (A) is what a cheap motel
and (D) are not mentioned. may not offer. Choice (C) is the
167. (D) The results discussed only address advantage of renting a room from a local
the Baltic market and should not be family. In choice (D), the word bus is
generalized to other markets. Choices mentioned, but in different contexts.
(A) and (B) contradict the last statement 177. (A) The Hoekland Foundation is an
that says the finding is only true for the economic development institute. Choice
Baltic stock market. Choice (C) is (B) is incorrect because the foundation is
incorrect because the findings are based nonpolitical. Choice (C) is not
on the Baltic markets. mentioned. Choice (D) associates
168. (B) Since JM Drilling Fluids lowers the investment firm with the foundation’s
cost of producing oil, the advertisement economic focus.
is probably intended for the oil industry. 178. (B) The information focuses on the
Choices (A) and (C) are not logical. Foundation’s purpose. Choice (A) is not
Choice (D) associates manufacturing with mentioned. Choice (C) is incorrect
technology and production. because although economic briefings are
169. (C) The company improves the given, dates for them are not. Choice (D)
technology of drilling. Choice (A) is not mentioned.
associates research consultants and 179. (C) A secondary school advisor would
Copyright © 2012 by Pearson Education, Inc. Permission granted to reproduce for classroom use.
industry experts. Choice (B) associates probably have the degree and experience
drilling equipment with drilling fluids. qualifications for this position. Choice
Choice (D) confuses promoting shale (A) associates economics instructor with
production and increasing the performance knowledge of island economies. Choice (B)
of shale inhibition. associates psychotherapist with counseling
170. (C) The EU is liberalizing its aviation experience. Choice (D) associates
industry. Choices (A), (B), and (D) are all postgraduate student with postgraduation
mentioned but are not the main idea of job placement opportunities.
the passage. 180. (A) Employment advertisements often
171. (B) Airport taxes are not mentioned. appear in newspapers. Choices (B), (C),
Choices (A), (C), and (D) are all methods and (D) are not logical.
the EU used to diminish the U.S. market. 181. (B) In his e-mail to Hugh Clifford,
172. (C) Foreign airlines are gaining market Jackson Donaldson writes I guess that’s
strength. Choices (A), (B), and (D) are not your job when you own a sprinkler company.
true statements. Choice (A) confuses the name of Mr.
Donaldson’s business and the police who
173. (D) Travelers would be the most will enforce the watering restrictions.
interested in inexpensive hotel rooms. Choices (C) and (D) are not mentioned.
Choices (A), (B), and (C) are not the
intended audience.
182. (B) From the date on the telephone 189. (B) In her letter, Barbara Reynolds
message we know that April 12 is a complained about the fact that there was
Friday. The watering restrictions begin no manager on duty last week. Choice
on April 15, a Monday. Choice (A) (A) is not mentioned. Choice (C) is
confuses the date of the telephone incorrect because employees are allowed
message. Choice (C) is when Mr. to have a night off with proper
Donaldson will be able to talk to Mr. arrangements. Choice (D) is incorrect
Clifford. Choice (D) is when Mr. because even though Barbara mentions
Donaldson wants to get together with her low pay, she doesn’t say that it was
Mr. Clifford. the cause of the problem.
183. (C) Mr. Donaldson writes I’m not really 190. (B) Barbara sent a letter of resignation.
comfortable writing our security code in an Choice (A) is what she did on several
e-mail. Choices (A), (B), and (D) are all nights during the previous week. Choice
things that he says he will do. (C) is what she did on her night off.
184. (B) According to Mr. Clifford’s message, Choice (D) confuses the apology she
the fine for the first offense is $50. Choice made to George and Maria Rhodes.
(A) is not mentioned. Choice (C) is the 191. (D) The mayor will speak about car theft,
fine for additional offenses. Choice (D) a type of crime. Choices (A) and (B) are
combines the two different fines. topics other speakers will address.
185. (B) Mr. Donaldson agrees to Mr. Choice (C) associates parking with car.
Clifford’s suggestion of going bowling. 192. (D) He wants to change time slots with
Choice (A) associates football with game. Council Member Jones, who was
Choice (C) is the job Mr. Clifford will do originally scheduled to be the second
on Monday. Choice (D) confuses meeting speaker. Choice (A) is the opposite of
for dinner with have lunch. what he wants to do—leave the meeting
186. (C) Maria was angry because a staffing early. Choice (B) is Council Member
problem caused several customers to Jones’s topic, but Morris doesn’t want to
walk out of the restaurant. Choice (A) is change topics. Choice (C) is incorrect
incorrect because the customers were because he would speak after the mayor
if he didn’t change the schedule.
Copyright © 2012 by Pearson Education, Inc. Permission granted to reproduce for classroom use.
195. (B) He wants to pick up his relatives at 198. (C) The installation took 6 hours longer
the airport. Choice (A) is what he will do than the originally estimated 15 hours.
the following day. Choice (C) is Choice (A) is the amount of extra time
associated with going to the airport. installation took. Choice (B) is the
Choice (D) is something he will have to originally estimated time. Choice (D) is
do before March 18. confused with the cost of some of the
196. (A) All flooring comes with a one-year flooring.
guarantee, so no extra payment is 199. (C) Mr. Moore writes that his secretary
required. Choice (B) is the cost of a ten- faxed the invoice. Choices (A), (B), and
year extended warranty. Choice (C) is the (D) are not mentioned.
amount the company is charged for labor
200. (B) The estimate is dated May 1, and the
costs. Choice (D) is the cost of 300 square
work was completed May 4, the day
feet of hardwood flooring.
before the secretary faxed the invoice.
197. (B) Mr. Moore’s letter says The amount Choice (A) is the amount of labor time
you sent was for the estimate that we issued the installation took, but it was
before doing the work. Choice (A) is completed over several days time.
incorrect because it is never mentioned Choice (C) is not mentioned. Choice (D)
that the company wanted to buy a is the amount of time between the date of
warranty. Choice (C) is incorrect because the estimate and the date of Mr. Moore’s
no problem with the materials is letter.
mentioned. Choice (D) is a mistake made
by the flooring company.
Copyright © 2012 by Pearson Education, Inc. Permission granted to reproduce for classroom use.
ANSWER KEY
PRACTICE TEST THREE
PART 1 (PAGES 301–306) 6. (A) The photo shows bookshelves in
what is probably a library. Choice (B)
1. (D) The photo shows a view of city
confuses similar-sounding words books
buildings with a baseball diamond in the
and cooks and reading and eating. Choices
foreground. Choice (A) is incorrect
(C) and (D) mention people you would
because it is daytime. Choice (B) is
see at a library (librarian, student) but are
incorrect because there are no people on
incorrect because there are no people in
the baseball diamond. Choice (C) uses
the picture.
the word building as a verb rather than a
noun and confuses similar-sounding 7. (C) A doctor writes on a clipboard while
words tall and mall. he talks with a patient. Choice (A)
confuses similar-sounding words writing
2. (B) A woman is holding a telephone
and riding. Choice (B) confuses similar-
receiver to her ear and listening. Choices
sounding words talking and walking.
(A), (C), and (D) all use the word phone
Choice (D) is incorrect because the
but describe actions that the woman is
patient is sitting up, not lying down.
not doing.
8. (D) A driver is sitting in a car examining
3. (A) A man is standing beside some tables
a dollar bill. Choice (A) uses the word
that are set with plates and napkins for a
car, but there is no mechanic in the
formal meal. Choice (B) mentions the
picture. Choice (B) confuses similar-
man but he is not serving. Choice (C) is
sounding words car and jar. Choice (C) is
incorrect because there is no food in the
incorrect because the man is wearing, not
picture. Choice (D) is incorrect because
waving, his hat.
there are no guests visible; the only
9. (A) A woman is sitting on a plane near
Copyright © 2012 by Pearson Education, Inc. Permission granted to reproduce for classroom use.
12. (A) This is a logical answer to the 22. (A) This is a logical response to the
question. Choice (B) confuses the question about the post office. Choice (B)
meaning of the word work. Choice (C) confuses similar-sounding words open
confuses same-sounding words where and pen. Choice (C) associates post office
and wear. with stamps.
13. (B) Asking for a pen is a logical response 23. (B) I missed the train is a logical
to a request to sign a document. Choice explanation for being late to a meeting.
(A) confuses the meaning of the word Choice (A) confuses similar-sounding
sign. Choice (C) confuses similar- words meeting and eating. Choice (C)
sounding words need and read and repeats the word late.
repeats the word document.
24. (A) On the next block answers the
14. (C) On my desk answers the question question How far? Choice (B) confuses
Where? Choice (A) confuses telephone similar-sounding words far and fare.
with the similar-sounding phrase tell Choice (C) would answer the question
them at home. Choice (B) repeats the word How often?
telephone.
25. (A) Sandwiches and ice cream is what was
15. (A) Twenty answers the question How served for lunch. Choice (B) would
many? Choice (B) repeats the words answer the question Where? Choice (C)
people and workshop. Choice (C) confuses would answer the question When?
workshop with work in this shop.
26. (B) Mary answers the question Who?
16. (C) Some colleagues answers the question Choice (A) repeats the word answer.
Who? Choice (A) would answer When? Choice (C) confuses similar-sounding
Choice (B) would answer How? words phone and loan.
17. (B) Two weeks answers the question How 27. (C) Near the airport answers the question
long? Choice (A) confuses away with the Where? Choice (A) associates car with
similar-sounding word day. Choice (C) driving. Choice (B) repeats the word car.
confuses away with the similar-sounding
28. (B) The morning train answers the
word stay.
question Which train? Choice (A)
18. (B) Let’s eat here is a logical response to a confuses similar-sounding words arrive
Copyright © 2012 by Pearson Education, Inc. Permission granted to reproduce for classroom use.
comment about a good restaurant. and five. Choice (C) associates train with
Choice (A) confuses similar-sounding tickets and station.
words restaurant and rest. Choice (C)
29. (A) This is a logical response to a
confuses similar-sounding words
comment about the color of the walls.
restaurant and best.
Choice (B) confuses similar-sounding
19. (C) Five or six years answers the question words walls and calls. Choice (C)
How Long? Choice (A) confuses similar- confuses similar-sounding words color
sounding words long and strong. Choice and collar.
(B) would answer the question How well?
30. (B) Last month answers the question
20. (A) This is a logical response to a remark When? Choice (A) confuses the meaning
about the size of the office. Choice (B) of the word last. Choice (C) repeats the
confuses similar-sounding words small word see.
and tall. Choice (C) confuses similar-
31. (C) Sunny and clear answers the question
sounding words quite and quiet.
about the weather. Choice (A) repeats the
21. (C) A hundred and fifty dollars answers the words report and tomorrow. Choice (B)
question How much? Choice (A) confuses homonyms weather and whether
associates hotel with reservations. Choice and repeats the word tomorrow.
(B) repeats the phrase stay at that hotel.
32. (C) Downtown answers the question 42. (C) They will meet at 9:00 because the
Where? Choice (A) repeats the word man can’t meet earlier. Choice (A) is the
afternoon. Choice (B) confuses similar- time the woman wants to meet. Choice
sounding words afternoon and room and (B) confuses similar-sounding words
repeats the name Mrs. Kim. late and eight. Choice (D) confuses
similar-sounding words then and ten.
33. (A) Mr. Brown answers the question
Who? Choice (B) confuses similar- 43. (B) The man has to stay at the office and
sounding words hired and tired. Choice work on a report. Choice (A) is confused
(C) repeats the word position. with the woman saying she is tired of
cooking. Choice (C) confuses similar-
34. (A) Tomorrow answers the question
sounding words home and phone. Choice
When? Choice (B) confuses related words
(D) repeats the word report.
available and avail. Choice (C) repeats the
name Mr. Jones and confuses similar- 44. (A) The man says that they are meeting
sounding words available and able. with the clients. Choice (B) is confused
with the topic of the meeting—the
35. (B) Some books answers the question
accounts. Choice (C) is confused with the
about what was in the package. Choice
mention of the clients’ offices being
(A) repeats the word package. Choice (C)
painted. Choice (D) associates money
associates package with the word
with bankers.
delivered.
45. (D) The man says that they will meet on
36. (A) The second speaker offers the use of
Friday. Choice (A) is when the clients
a stapler, a logical response to the
called to change the location of the
statement about a missing stapler. Choice
meeting. Choices (B) and (C) sound
(B) uses the related word staples. Choice
similar to today.
(C) repeats the word stapler.
46. (B) The woman says that she reserved the
37. (B) To a meeting answers the question
conference room for the meeting. Choice
Where? Choice (A) repeats the phrase this
(A) is the location of the clients’ office.
morning. Choice (C) repeats the words go
Choice (C) is associated with the lunch
and morning.
that will be served. Choice (D) confuses
38. (C) It’s John’s answers the question about
Copyright © 2012 by Pearson Education, Inc. Permission granted to reproduce for classroom use.
50. (A) The man asks the woman to 57. (A) The woman says she’d rather leave in
recommend some books to take on his the morning. Choice (B) is when she
vacation. Choice (B) confuses similar- doesn’t want to leave. Choice (C)
sounding words reading and meeting. confuses similar-sounding words leaving
Choice (C) confuses the meaning of the and evening. Choice (D) confuses similar-
word book. Choice (D) confuses the sounding words right and night.
meaning of the word light.
58. (B) The man says that he will call the
51. (D) The woman suggests talking to her airline to change the plane ticket. Choice
boss because she knows a lot about (A) confuses similar-sounding words
books. Choice (A) confuses similar- plane and train. Choice (C) confuses
sounding words more and store. Choice similar-sounding words us and bus.
(B) confuses similar-sounding words time Choice (D) is not mentioned.
and online. Choice (C) is confused with
59. (D) The woman offers to make the
the man’s saying that he wants some
copies. Choice (A) is the person who
relaxing books.
wrote the document to be copied. Choice
52. (D) The man says he is leaving on (B) is out sick. Choice (C) is the person
Sunday. Choice (A) is when the man who wants the copies made.
wants to find the books. Choice (B) is
60. (B) The man asks for 500 copies. Choice
when the woman’s boss will be available.
(A) is the number he starts to ask for, but
Choice (C) sounds similar to the correct
then changes his mind. Choice (C)
answer.
confuses similar-sounding words sick
53. (B) The woman says that the conference and six. Choice (D) confuses similar-
will last for three days and the man sounding words time and nine.
confirms this. Choice (A) confuses
61. (A) The man explains that the copies will
similar-sounding words to and two.
be part of a fund-raising packet that he is
Choice (C) confuses similar-sounding
sending to donors. Choice (B) is confused
words for and four. Choice (D) is not
with the meeting that the woman will go
mentioned.
to after lunch. Choice (C) confuses lunch
54. (A) The man says some rooms have been with the related word luncheon. Choice
Copyright © 2012 by Pearson Education, Inc. Permission granted to reproduce for classroom use.
65. (C) The woman mentions the rain. Choice 73. (A) By pressing 1, customers can place an
(A) confuses similar-sounding words not order. Choice (B) is what can be done by
and hot. Choice (B) confuses similar- pressing 3. Choice (C) is what can be
sounding words slow and snow. Choice done by pressing 2. Choice (D) is
(D) repeats the word clear. confused with the announcement that the
menu has recently changed.
66. (D) The man suggests the subway and
the woman agrees. Choice (A) is what the 74. (C) The announcer says that the bridge is
woman says will be impossible. Choice closed due to flooding. Choice (A) is
(B) confuses similar-sounding words far incorrect because the announcer says
and car. Choice (C) confuses similar- that no accidents have been reported.
sounding words rain and train. Choice (B) is what is happening on
Highway 11, not on the bridge. Choice
67. (B) The woman asks the man to carry her
(D) is not a problem according to the
suitcase. Choices (A) and (C) are things
report.
the man mentions. Choice (D) is
confused with the mention of the subway 75. (B) The announcer says that today there
schedule. is sunny weather and a cloudless sky.
Choice (A) confuses related words
68. (B) The woman will arrive on Monday,
cloudless and cloudy. Choice (C) is how
stay for Monday and Tuesday night, and
the weather was yesterday. Choice (D) is
leave on Wednesday. Choice (A) sounds
not mentioned.
similar to the correct answer. Choice (C)
is the second night she will be at the 76. (B) The announcer says that the next
hotel. Choice (D) is when she will leave. traffic report will be at 6:00. Choice (A) is
confused with rush hour. Choice (C) is
69. (A) The woman will stay Monday and
confused with Highway 11. Choice (D) is
Tuesday nights. Choice (B) is confused
when the bridge may reopen.
with the number of rooms in a suite.
Choice (C) is confused with the number 77. (A) Lakeland is a company that offers
of people that can sleep in a suite. Choice cruises of Lake Pine, along with themed
(D) confuses similar-sounding words fine learning programs about the lake. Choice
and nine. (B) is confused with the mention of
Copyright © 2012 by Pearson Education, Inc. Permission granted to reproduce for classroom use.
80. (C) The announcer uses the word museum 88. (A) Some people are concerned that the
two times as well as mentioning the factory will make their neighborhood an
galleries. Choice (A) associates packages unattractive place to live. Choice (B) is
with post office. Choice (B) is confused confused with the concern about the
with the mention of the coat check room. effect the factory will have on property
Choice (D) associates check with bank by prices. Choice (C) contradicts
confusing its meaning. information in the report. Choice (D)
81. (D) The announcer says that the time is repeats the word construction.
6:45 and that the museum will close in 15 89. (B) The announcer says: Subway number 7
minutes. Choices (A) and (B) sound is due to arrive in the station in two minutes.
similar to the current time. Choice (C) is Choice (A) repeats the word schedule.
the current time. Choice (C) repeats the word weather.
82. (A) Visitors are asked to use the main Choice (D) is where the subway train is
entrance rather than the side or basement headed.
entrances. Choice (B) repeats the words 90. (A) The announcer says the train will
packages and desk. Choice (C) is confused make no stops. Choice (B) is confused
with We hope you have enjoyed your visit. with the number of the train. Choice (C)
Choice (D) repeats the words proceed and confuses similar-sounding words late
basement. and eight. Choice (D) sounds similar to
83. (C) The announcer says that a map is the number of the train.
posted by the door to help people find 91. (B) The announcer says to stand back from
the workshops. Choice (A) is in the the edge of the platform to allow arriving
packets. Choice (B) repeats the word passengers to exit. Choice (A) repeats the
packet. Choice (D) is confused with word stand. Choices (C) and (D) are what
posted. people are asked not to do.
84. (A) Lunch will be served in a restaurant 92. (C) An orchestra will play, so it is a
on the ground floor. Choices (B) and (C) concert. Choice (A) confuses the meaning
are where the workshops will be held. of the word play. Choice (B) is associated
Choice (D) is confused with third annual with the location of the event near the
Copyright © 2012 by Pearson Education, Inc. Permission granted to reproduce for classroom use.
96. (C) According to the report, tomorrow will 104. (C) The subject of the sentence, money, is
be warm and sunny. Choice (A) is how the passive—it does not perform the action
weather is this morning. Choice (B) is but receives it. Therefore, the past
confused with the similar sounding word participle is needed to complete the
school. Choice (D) is confused with the passive voice verb. Choices (A) and (B)
similar-sounding word easy. are active present tense forms of the
verb. Choice (D) is present participle.
97. (D) This will be the high temperature
according to the report. Choice (A) 105. (B) To turn up the volume means to
sounds similar to 50, the low increase it. Choices (A), (C), and (D) are
temperature tonight. Choice (B) sounds all words that can be used with turn to
similar to the correct answer. Choice (C) create phrasal verbs, but their meanings
is the low temperature tonight. would not fit the context.
98. (C) The speaker will talk about her book, 106. (A) The base form of the verb is used for
How to Start a Small Business Without the imperative, to give a command or
Financial Ruin. Choice (A) repeats the make a request. Choice (B) is present
word book. Choice (B) repeats the word tense. Choice (C) is infinitive. Choice (D)
newspaper, where the speaker has is present participle or a gerund.
worked. Choice (D) repeats the word 107. (D) An adverb is used to modify the verb
financial. asks. Choice (A) is a noun or verb. Choice
99. (C) The announcer says that the speaker (B) is a verb. Choice (C) is an adjective.
will be available to sign books. Choices (A) 108. (C) A comparative adverb is used to
and (B) happened before the lecture. complete the comparison with than.
Choice (D) will happen next week. Choice (A) is an adjective. Choice (B) is
100. (B) The announcer says that the regular an adverb but not a comparative form.
lecture schedule will continue in two Choice (D) is a superlative form and
weeks. Choice (A) is when there will be a cannot be used with than.
film. Choice (C) confuses similar- 109. (A) Who is used as a relative pronoun to
sounding words two and few. Choice (D) introduce an adjective clause, taking the
hasn’t been scheduled yet. position of the subject of the clause.
Copyright © 2012 by Pearson Education, Inc. Permission granted to reproduce for classroom use.
113. (D) This is the future perfect tense, used 125. (D) Profits means money earned by a
to describe an action that will be business. Choices (A), (B), and (C) are
completed in the future. Choice (A) is words related to money but don’t fit the
simple past. Choice (B) is simple future, meaning of the sentence.
and Choice (C) is present continuous.
126. (A) Should is a modal so it is followed by
114. (C) At is used with an exact time of day. a base form verb. Choice (B) is present
Choices (A), (B), and (D) are not used to tense. Choice (C) is infinitive. Choice (D)
indicate an exact hour. is a gerund or present participle.
115. (A) The plural verb look agrees with the 127. (D) A present tense unreal conditional
plural subject books. Choices (B) and (D) uses the conditional form, would + base
are singular verb forms. Choice (C) is form verb, in the main clause. Choice (A)
infinitive. is present tense. Choice (B) is past tense.
116. (B) This is the main verb of the sentence Choice (C) is future tense.
and agrees with the singular subject, 128. (C) An adjective is needed to modify the
phone. Choice (A) is a verb but does not noun renovations. Choice (A) is a verb.
agree with the subject. Choice (C) is a Choices (B) and (D) are nouns.
noun. Choice (D) is an adjective.
129. (B) Remote means far away. Choices (A),
117. (C) And is used to add similar (C), and (D) have meanings that don’t fit
information. Choice (A) is used to the context.
indicate a choice. Choices (B) and (D) are
130. (A) After introduces the time clause.
not used to introduce a clause.
Choice (B) could introduce a time clause
118. (B) This singular verb form agrees with but has the wrong meaning for this
the singular subject Nobody. Choices (A), sentence. Choice (C) introduces a result.
(C), and (D) are plural verb forms so they Choice (D) cannot be used to introduce a
do not agree with the subject. clause.
119. (D) The verb suggest is followed by a 131. (B) Although introduces a contradiction.
gerund. Choice (A) is base form or Choice (A) introduces a reason. Choices
present tense. Choice (B) is infinitive. (C) and (D) cannot be used to introduce a
Choice (C) is past tense. clause.
Copyright © 2012 by Pearson Education, Inc. Permission granted to reproduce for classroom use.
120. (A) A base form verb follows a modal. 132. (D) This is a negative imperative form
Choices (B) and (C) are nouns. Choice used to make a request. Choice (A) is
(D) is an adjective. infinitive. Choice (B) is present
121. (B) An unreal past tense conditional continuous. Choice (C) is a gerund.
requires the past perfect verb form in the 133. (C) The plural verb are agrees with the
if -clause. Choice (A) is simple past tense. plural subject names. Choices (A), (B),
Choice (C) is present perfect tense. and (D) are all singular forms.
Choice (D) is past continuous.
134. (C) The verb avoid is followed by a
122. (B) The verb agree is followed by an gerund. Choice (A) is base form or
infinitive verb. Choice (A) is past tense. present tense. Choice (B) is past tense.
Choice (C) is present tense. Choice (D) is Choice (D) is infinitive.
a gerund or present participle.
135. (D) The present perfect tense is used to
123. (A) The singular verb has gone agrees describe an action that started in the past
with the singular subject rent. Choices and continues to the present. Choice (A)
(B), (C), and (D) are all plural forms. is simple present tense or base form.
124. (C) Because introduces a reason. Choice Choice (B) is past tense. Choice (C) is
(A) introduces a condition. Choice (B) future.
introduces a contradiction. Choice (D)
introduces a result.
136. (C) So introduces a result. Choices (A) 147. (B) Complain means find a problem with
and (D) introduce a reason. Choice (B) something. Choices (A), (C), and (D) look
introduces a contradiction. similar to the correct answer but have
meanings that don’t fit the context.
137. (A) Remain means stay. Choices (B), (C),
and (D) look similar to the correct 148. (D) Ideal means best or perfect. A 10- or 15-
answer but have meanings that don’t fit minute walk from the office is the best
the context. location because it gives the opportunity
for exercise. Choices (A), (B), and (C)
138. (B) Drop is the main verb of the clause.
don’t fit the meaning of the passage.
Choices (A), (C), and (D) are not verbs.
149. (A) This is future perfect tense used to
139. (C) Put off means postpone. Choices (A),
describe an action that will be completed
(B), and (D) can also be used with put to
before another action in the future. Here
form phrasal verbs, but the meanings
it means that you will complete a certain
would not fit the sentence.
amount of exercise before you arrive at
140. (D) The adverb expectantly modifies the your desk. Choice (B) is present
verb waited. Choice (A) is a verb. Choice continuous tense. Choice (C) is past
(B) is an adjective. Choice (C) is a noun. perfect. Choice (D) is simple future.
PART 6 (PAGES 317–320) 150. (C) Expansion means growth. This is the
correct meaning because we know from
141. (C) Providing means giving or making the next sentence that the company is
available. Choices (A), (B), and (D) look growing. Choices (A), (B), and (D) have
similar to the correct answer but have meanings that don’t fit the context.
meanings that don’t fit the context.
151. (B) This is the past participle used to
142. (B) The second person possessive form the present perfect verb have
adjective your refers to the reader of the recently completed. Choice (A) is present
advertisements, whose legal needs the tense or base form. Choice (C) is present
law firm would like to help with. Choice participle or a gerund. Choice (D) is a
(A) would refer to the advertisers, that is, noun.
the law firm, which doesn’t make sense.
Choices (C) and (D) are third person 152. (A) This is a first person possessive
Copyright © 2012 by Pearson Education, Inc. Permission granted to reproduce for classroom use.
155. (D) The workshop begins at 9:00 and lasts 163. (D) The trip to Australia involves seven
for three hours, so it ends at 12:00. Choice days of bicycle riding plus four days in
(A) is confused with the length of the Melbourne, for a total of eleven days.
workshop. Choice (B) is the time the Choice (A) is the number of days in
workshop begins. Choice (C) is confused Melbourne only. Choice (B) is the
with the date of the workshop. number of days on bicycles only. Choice
156. (A) They bought a retail business which (D) is the number of days for the Alaskan
sells gifts, souvenirs, books, and cruise.
magazines. Choice (B) is associated with 164. (B) The trip to Italy includes hiking in the
the coffee and snacks they will sell, but Alps, that is, walking in the mountains.
these will not be the main part of the Choice (A) includes looking at
business. Choice (C) is where they got mountains from a ship. Choice (C) is a
their loan. Choice (D) is what the trip to a baseball game. Choice (D)
building was originally built for. includes walking around a town.
157. (B) They purchased it at the end of last 165. (A) A price chart is on the last page of the
year. Choice (A) is confused with when brochure. Choice (B) refers to the trip to
they will open the business, next month. the baseball game. Choice (C) refers to
Choices (C) and (D) are confused with the the trip to the theater—presumably, the
recent history of the business: which has history will be presented during the trip,
opened and closed under new management not in the brochure. Choice (D) refers to
three times in the past five years. the baseball game trip.
158. (D) They interviewed local residents to 166. (A) The job involves working with
find out what the customers would want. medical records and patients, and
Choice (A) is one thing they will do as experience in a medical office is
part of the business. Choice (B) confuses mentioned, so it must have been placed
the meaning of the word housed—the by a doctor. Choice (B) is associated with
building once housed, or contained, a the mention of vacations, referring to a
factory. Choice (C) repeats the word benefit of the job. Choice (C) repeats the
management. word software, which refers to a skill the
applicant should have. Choice (D)
Copyright © 2012 by Pearson Education, Inc. Permission granted to reproduce for classroom use.
170. (B) Customers and clients can park at no 178. (A) Beverages means drinks. Choices (B),
charge, that is, for free, by presenting a (C), and (D) are all things that are served
validated ticket to the attendant. Choice at cafés but are not the correct meaning.
(A) is what to do if you have questions.
179. (B) The advertisement mentions
Choice (C) is where compact cars can be
breakfast and lunch. Choices (A) and (C)
parked. Choice (D) refers to places in the
are incorrect because the café doesn’t
garage where visitors can park.
serve only one of these meals. Choice (D)
171. (B) Funds deposited before 5:00 P.M. are is incorrect because dinner, an evening
available on the next business day. meal, is not mentioned and the café
Choices (A), (C), and (D) don’t follow closes at 2:00.
this formula.
180. (C) The three-course special costs $12
172. (A) Everyday that is not Sunday or a and with the ten percent delivery charge,
holiday is a business day. Choices (B), the total is $13.20. Choice (A) is the price
(C), and (D) don’t fit this description of the two-course special. Choice (B) is
173. (D) The last paragraph tells us that funds the price of the three-course special
drawn in foreign banks are usually without the delivery charge. Choice (D)
available in ten business days. Choice is the price of the picnic basket.
(A) is how long it takes a regular deposit 181. (C) Louis sent Sandra the itinerary for
under $5,000 to clear. Choice (B) is not review, and Sandra wrote back
mentioned. Choice (C) is how long it requesting some changes. Choice (A) is
takes deposits over $5,000 to clear. associated with the discussion of hotels,
174. (C) The information explains that the but no recommendations are requested.
classes are designed for business Choice (B) is incorrect since Louis sent
executives, that is, people already the itinerary to Sandra, we can assume
working in business. Choice (A) is he knows about the trip. Choice (D) is
incorrect because it is for people already associated with the discussion of flights
working, not still studying. Choice (B) is and hotels, but prices aren’t mentioned.
confused with the accounting class 182. (B) Sandra is scheduled to attend a
offered. Choice (D) might be someone conference from Monday through
Copyright © 2012 by Pearson Education, Inc. Permission granted to reproduce for classroom use.
who would teach at the school. Wednesday. Choice (A) is something she
175. (C) The management class lasts for four will do but is not the reason for her trip.
weeks, and tuition for four-week classes Choices (C) and (D) are things she will
is $500. Choices (A) and (B) are confused do in Montreal.
with the refund percentages. Choice (D) 183. (D) Sandra asks Louis to get her a room
is the cost of an eight-week class. at the Knickerbocker Inn. Choice (A) is
176. (D) A 100 percent refund is available where Sandra is scheduled to stay, but
before the first class, which in this case she says she won’t stay there. Choice (B)
takes place on the evening of June 17. is where the conference will be held.
Choices (A) and (B) are registration Choice (C) is where she will stay in
dates. Choice (C) is the first day of the Montreal.
management class. 184. (C) Sandra will be in Montreal
177. (B) The information tells us that books Wednesday, Thursday, and Friday nights.
are for sale in the admissions office. She was originally scheduled for
Choice (A) is incorrect because the Thursday and Friday nights, but says she
bookstore is closed for the summer. wants to leave New York a day earlier
Choice (C) is incorrect because books than planned. Choices (A), (B), and (D)
have to be purchased before the first don’t match the plans Sandra is making.
class. Choice (D) is a way to register for
classes but not to buy books.
185. (A) Sandra tells Louis that she plans to 193. (C) The company needs people to help
invite her friends to dinner on Friday. take orders by phone. Choice (A) is
Choices (B), (C), and (D) are things she confused with the ad placed by the
will do on Thursday. Skilled Office Temps company. Choice (B)
186. (B) Ms. Rosen thanks Ms. Ogden for is incorrect because the temporary
organizing her office. Choice (A) is a workers will not fulfill orders (package
similar business but is incorrect because and mail them) but take them. Choice (D)
the Ophelia Company does not decorate, is one of the services offered in the ad.
it organizes. Choice (C) is associated 194. (C) Mr. Murray asks for workers with
with the furniture that was ordered as customer service experience. Choices (A)
part of the organizing process. Choice and (B) are what he says is not necessary.
(D) is associated with the invoice. Choice (D) is confused with the mention
187. (D) Payment is due within 30 days of the of the newspaper, where Mr. Murray saw
invoice date, which is May 1. Choice (A) the ad.
is the invoice date. Choice (B) is the date 195. (D) Inter Metro will hire two workers for
of the letter. Choice (C) is not mentioned. 120 hours each, or 240 hours total, at $20
188. (B) Ms. Rosen believes she should pay for an hour ($4,800), with a ten percent
only three hours because she thinks the discount because the assignment is longer
first hour should be free. Choice (A) is than one week. Choice (A) is the charge
the price for one hour. Choice (C) is the for one worker without the discount.
amount billed on the invoice. Choice (D) Choice (B) is the charge for two workers
is the total due on the invoice. without the discount. Choice (C) is the
charge for one worker with the discount.
189. (A) Three bookshelves were ordered, but
Ms. Rosen only wants two. Choice (B) is 196. (B) Li is making the arrangements for a
incorrect because Ms. Rosen says the meeting next week. Choices (A) and (C)
bookshelves fit in the space. Choices (C) are other types of events that the catering
and (D) are incorrect because Ms. Rosen company caters. Choice (D) is confused
says she likes the color and style. with the location of the meeting in the
conference room.
190. (C) Ms. Rosen asks for a revised invoice.
Copyright © 2012 by Pearson Education, Inc. Permission granted to reproduce for classroom use.
Choice (A) is what she offers to give Ms. 197. (D) Green salad adds $30 to the price.
Ogden. Choice (B) is incorrect because Choices (A), (B), and (C) are all included
Ms. Rosen thanks Ms. Ogden for her help in the $250 price.
but doesn’t ask for more. Choice (D) is 198. (A) Mr. Brown says that the cost seems
confused with the discussion of the new fair to him. Choice (B) is not mentioned.
furniture that Ms. Rosen already has. Choice (C) is how Li thinks Mr. Brown
191. (A) Mr. Murray writes that he saw the ad might feel. Choice (D) is the opposite of
two days ago, and his e-mail is dated what Mr. Brown says.
November 8. Choice (B) is the date of the 199. (C) Li is planning for 25 people, and then
e-mail. Choices (C) and (D) are the dates Mr. Brown says that five more will attend.
for beginning and ending the temporary Choice (A) is five less than the number Li
work assignment. is planning for. Choice (B) is the number
192. (B) Mr. Murray writes that it is the busy Li is planning for. Choice (D) is not
season at his company. Choices (A), (C), mentioned.
and (D) are reasons suggested in the ad. 200. (C) The caterers said they would arrive at
11:30, and Mr. Brown wants them to
arrive an hour later than that. Choice (A)
is the original time they planned to arrive.
Choice (B) is the original time scheduled
for lunch to begin. Choice (D) is the new
time scheduled for lunch to begin.